mbe portion ubebb

126

Upload: others

Post on 20-May-2022

0 views

Category:

Documents


0 download

TRANSCRIPT

Page 1: MBE Portion UBEBB
Page 2: MBE Portion UBEBB

MBE Portion of The UBE Bridge Book

By Mo Hoosen

Page 3: MBE Portion UBEBB

Civil Procedure Introduction Civil procedure (CivPro) is governed by the Federal Rules of Civil Procedure (FRCP, “the Rules”). These are of varying importance but, for our purposes, the most relevant sections are Title II and Title III which deal with service and jurisdiction respectively. The Rules govern proceedings in the US district and appellate courts. Their overriding objective is to “secure the just, speedy, and inexpensive determination of every action and proceeding”.1 This section will deal with the following facts: Abby, a resident of State A, owns and runs a marketing company Abby’s PR. Abby holds intellectual property protection over her ad campaigns. They are very successful in State A because all of Abby’s clients are in State A. Benny, who lives in State B and runs their own company, decides to use Abby’s designs. Benny first came across Abby’s designs because Eric, who lives in State A and is familiar with Abby’s work, showed them to Benny and said, “I’ll sell you a licence to use these designs for your own work”. Abby decides to file a suit against Charlie for infringement.

1 Fed. R. Civ. P. 1

Page 4: MBE Portion UBEBB

Jurisdiction Jurisdiction is the power to deal with legal issues. A court needs three things to validly decide a case:

• Personal jurisdiction, • Subject-matter jurisdiction, and • Venue

Personal jurisdiction (PJ) is the power a court has to hear a case against a person (or corporation). This differs from subject-matter jurisdiction (SMJ), which is the power the court has to address a specific legal issue, or a “subject”. The doctrine of minimum contacts is important because it highlights the minimum requirements an individual must possess in order for the court to have personal jurisdiction over them. That is to say, a defendant in State B can’t generally be sued for an action that occurred in State A. However, personal jurisdiction can be broken down into two arms: general or specific jurisdiction. General jurisdiction eliminates the need for minimum contacts on one of three bases. The Defendant gives or has:

1. Consent, 2. Presence, or 3. Domicile

1. Consent Consent by the Defendant means that they consent to be sued in the state where the summons is. Example (CivPro: Consent) Simon causes a car accident in State B and Ian is injured. If Simon consents to be sued by Ian, the court has general jurisdiction. Consent can be express or implied. In the example above, the consent was express. However, implied consent can take the form of Simon ceasing to object to being sued in State B, which is satisfactory for purposes of establishing general jurisdiction under consent. 2. Presence The Presence of the Defendant in the state gives the court general jurisdiction. This is established by serving the Defendant in that state. It can also be established if the Defendant has a continuous or systematic presence in the state. Example (CivPro: Presence) Simon causes a car accident in State B, where he regularly travels for work from State A, and Ian is injured. The court State B has general jurisdiction over the case because Simon has a continuous presence there. 3. Domicile If the Defendant is domiciled in a state, the court has general jurisdiction over the case. Domiciled is not the same as residence. Domiciled is an intent to live somewhere indefinitely, whereas a resident can be a temporary situation.

Page 5: MBE Portion UBEBB

Example (CivPro: Domicile) Simon causes a car accident in State B, where he works, on a weekend trip from State A, his home. The court will not have general jurisdiction based on domicile because his home is State A and he does not consider State B to be his domicile. If general jurisdiction is present, there is no need to examine contacts with the state, and thus no need to examine if the suit emerges as a result of those contacts. Once general jurisdiction is established, the Defendant may be sued for anything at all. On the other hand, specific jurisdiction arises out of contacts with the state. In other words, the court in State A argues that it has jurisdiction over the Defendant because their activities in State A gave rise to the claim. Under International Shoe Co. v Washington, the Defendant must always have minimum contacts with the state so that it does not fall foul of “traditional notions of fair play and substantial justice”. In order to establish specific jurisdiction, states have “long-arm” statutes. In an exam, apply the following test:

1. Does the state have a long-arm statute? If no, then there is no specific jurisdiction. If yes,

2. Are the constitutional due process requirements (CDPR) met? If no, there is no specific jurisdiction, if yes, there is specific jurisdiction. The CDPRs are:

a. Minimum contacts, which are the bases for general jurisdiction, and b. Is it fair and reasonable for the Defendant to be sued in the forum state?

Example (CivPro: Minimum Contacts) Simon lives in State A and travels to State B for work. While driving home from work, just before crossing state lines, Simon meets in a car accident and injures Ian. State B has a long-arm statute for torts occurring in the state. In this case, Ian can sue Simon for the injury caused because Simon has minimum contacts within State B. Subject-matter jurisdiction is the threshold that distinguishes the matter of a case from being discussed at state court versus federal court. Only certain types of cases can be heard at federal court. SMJ can be divided into two arms: citizen diversity jurisdiction and federal question jurisdiction. Citizen diversity jurisdiction permits Plaintiffs to bring claims to federal court, but there are some requirements to establish standing:

1. The Plaintiff(s) and Defendant(s) cannot be citizens of same state, and 2. The claim must exceed $75,000.

(1) In establishing citizenship, the court relies on where the Plaintiff and Defendant are domiciled. If either of the parties are natural persons, their domicile is comprised of:

• Their residence, and • The subjective intent to make the state their permanent home.

If either of the parties are corporations, their domicile is comprised of: • The principal place of business, and • Any state where it is incorporated.

If either of the parties is an executor or personal representative (administration of estates), their domicile is comprised of:

• The citizenship of the decedent or the person being represented.

Page 6: MBE Portion UBEBB

(2) In establishing whether the claim exceeds $75,000, this can be as simple as loss incurred or based on damages alleged in good faith. However, if it is legally certain that the Plaintiff cannot recover the amount, the federal court can still hear the case and award injunctive relief or aggregate claims. Example (CivPro: Citizen Diversity Jurisdiction) Abby runs her very successful marketing business in State A. She has invested significant sums of money (north of $100,000) in developing cool new designs and slogans, which she owns completely. Charlie, who lives in State B, sees Abby’s promotions and decides to copy all of them. Can Abby sue Charlie? In the example above, Abby can sue Charlie in federal court because they both live in different states and the claim exceeds $75,000. However, there is one huge exception. If a Defendant is sued in state court, they cannot remove the case to federal court if the only basis for removal is diversity and the Defendant is sued in their home state. In effect, if Abby sues Charlie in State B, Charlie cannot motion for removal to federal court. However, if Abby sues Charlie in State A, Charlie can motion for removal to federal court as State A is not Charlie’s home state. Charlie can only motion for removal to federal court in the district where the state court case originally filed if:

• The federal court has SMJ, • All Defendants agree, • The Defendant is not a resident of the forum state, and • Removal is sought within 30-days of the service of the summons or receiving

the initial pleading What if the claim is not a monetary one? This where federal question jurisdiction comes into play. This asks: “Does the case at hand have an impact on a federal level?” To answer this question, apply the following rubric:

1. The claim must arise under: o Federal law, o The US Constitution, or o Any US treaty

2. The Plaintiff must be enforcing a federal right, 3. The federal question must be present on the face of a well pled complaint, and 4. The case must not involve domestic relations matters

To sum up, SMJ is an important aspect as to the meat of the case. To bring a parallel of sorts with the UK, imagine the federal court as analogous to the ECHR in that it only hears specific types of cases. In order words, it only hears cases which contain a certain type of subject-matter. There is an exception to the necessity of SMJ, which is called supplemental jurisdiction. This allows a party to bring a state claim to federal court even if it doesn’t meet the SMJ rubric. In order to exercise this, the claim must arise from a “common nucleus of operative fact”. In essence, they must derive from the same situation such that a Plaintiff would ordinarily expect to try them all in one judicial proceeding. Therefore, this is less of an exception and more of an add-on.

Page 7: MBE Portion UBEBB

Example (CivPro: Supplemental Jurisdiction) In addition to Charlie copying her designs, Abby also discovers that one of her employees, Dani, violated the non-compete clause in their contract and went to work for Charlie on a remote-work basis because they did not like how Abby spoke about Charlie. Ordinarily, violation of a non-compete clause is a matter for state court to decide, however, under supplemental jurisdiction, Abby can sue Dani for damages in federal court so long as it is tied to the primary matter: Charlie copying her designs. Supplemental jurisdiction can also arise if there is a nondiverse party in a diversity suit. Example (CivPro: Impleading) Charlie says to Abby, “I didn’t steal your designs, I bought them from Eric.” Eric, as it turns out, lives in State A – the same state as Abby. What can Abby do in relation to Eric? Abby’s claim in federal court is still against Charlie, but Charlie can implead Eric. Impleading means “bringing in” and follows the process of:

Note that there is no route for Abby to directly sue Eric in federal court. Charlie is saying to Abby, “If you’re going to sue me, I’m going to sue Eric,” and the matter as a whole will be dealt with in federal court. On a more nuanced note, impleading is not the same as a third-party claim. Impleading is where the initial Defendant (D1) admits that they are involved, but at fault on the basis of a third-party (D2). In the event that D1 denies any involvement, and pushes it to D2, D1 would file a motion to dismiss the suit. Example (CivPro: State a Third-Party Claim) If Charlie says to Abby, “You’ve got the wrong person, the person you’re after is Eric,” then he would seek to dismiss the suit and properly assert a third-party claim.

Abby (State A)

SuesCharlie

(State B)Sue

s

Eric (State A)

Page 8: MBE Portion UBEBB

Finally, there are times when the federal court can abstain from hearing a case if it may intrude upon the power of another court or if it awaits the outcome of a pending state court case. These are the abstention doctrines:

• Pullman Doctrine: the discretion to abstain when cases arise from an unsettled area of state law and the decision from state court may remove the need to decide the case federally;

• Younger Doctrine: the discretion to abstain from hearing constitutional challenges to state action when it would interfere with a state judicial proceeding;

• Colorado River Doctrine: the imposition to abstain when substantially the same federal and state litigations are pending; and

• Burford Doctrine: where abstention is only appropriate if federal adjudication would interfere with a state’s administration of a complex regulatory scheme.

Page 9: MBE Portion UBEBB

Service Service is the procedure by which a party to a lawsuit gives notice to initialise proceedings. The party bringing the suit is called the Plaintiff, and the party to whom the suit is against is called the Defendant. There are two aspects to service:

• Complaint, and • Summons

The complaint details the nature of the alleged offence, including who committed it, when it was committed, how long ago it was committed, etc. The summons contains the information pertaining to where the Defendant is being summoned, i.e. the venue, the date to appear, and who the Plaintiff and their lawyer are. Service must be done within 90 days of filing a suit. However, under Rule 4, service can be waived. Service can be waived by a Plaintiff by notifying the Defendant:

i. Be in writing and be addressed: a. To the Defendant, or b. To an officer, managing or general agent, or any other

agent authorised by appointment to receive service of process,

ii. Name the court where the complaint was filed; iii. Be accompanied by a copy of the complaint and other

requirements outlined in Rule 4 If the Defendant is within the US, the time limit to apply for default based on a waiver that was not responded is at least 30 days. This limit is at least 60 days if based outside the US. If a Defendant fails to respond, the court must impose expenses incurred in making service and the reasonable expenses of any motion required to collect those expenses. Service of proceedings against a person is done by delivering the complaint and summons (or a copy of the complaint and summons) one of four ways:

• State law methods, i.e. any method of service outlined in the laws of that state; • Agent, i.e. delivery to an agent authorised to receive service of process; • Individual, i.e. delivering it to the individual personally; or • Dwelling, i.e. leaving a copy of at the individual’s dwelling or usual place of

abode with someone of suitable age and discretion who resides there. Service of proceedings against a corporation is done by delivering the complaint and summons (or a copy of the complaint and summons) one of five ways:

• Same as an individual, or • Officer of the company

Service can be made by anybody who is at least 18 years old and not a party.

Page 10: MBE Portion UBEBB

Venue Venue is where the litigation takes place. Proper venue (PV) establishes the federal venue within the 50 states, whereas PJ establishes the jurisdiction to hear a case within a specific state. Because there are 50 states, but 94 federal districts, finding proper venue is a necessary pretrial step. Venue is proper in any district where:

• Any Defendant resides (if the Defendants are residents of the forum state), • Where a substantial portion of the claim/breach/omission occurred/property is

located, or • If neither, where any Defendant is subject to the court’s PJ.

Example (CivPro: Proper Venue 1) Abby decides to pursue her action against Charlie. She knows that Charlie committed the action in State B, so she can bring a claim to a federal district in State B. Example (CivPro: Proper Venue 2) Greg, who lives in State F, is driving with his friend Sam, who lives in State H, to go to New York. On the way there, they stop in New Jersey and drive over Louise’s foot. Louise, a resident and citizen of New Jersey, decides to claim against Greg and Sam. Where should she choose her venue? The federal rules state that a Plaintiff can only sue in the Defendant’s state if all Defendants are resident of the forum state. Because Greg and Sam do not live in the same state, and the incident occurred in New Jersey, Louise’s venue would be a federal district in New Jersey. The venue can be reallocated or transferred. If the venue was proper when the case was filed, the court may transfer it if:

• It is convenient for witnesses or in the interests of justice, and • The case could have initially been brought in the receiving court, i.e. the

receiving court has PJ and SMJ If the venue was improper when the case was filed, the court may:

• Transfer it to another federal district that has proper venue, or • Dismiss it for improper venue.

There may be instances where both venue and PJ are satisfied, but there is a better place to litigate and transfer is not possible. Facts (CivPro: Forum Non Conveniens) Louise, fresh off the win from her case, decides to go for a walk along the pier where she is once again struck by two tourists: Ron and Harry. Ron and Harry reside in Country A, the legal system of which is known to favour residents. If Louise files the suit in New Jersey, what can Ron and Harry do? A party seeking a would file a motion for forum non conveniens dismissal. Effectively, this allows them to seek a refiling of the case to Country A. Whether or not this would be successful is another matter.

Page 11: MBE Portion UBEBB

Finally, if the case pertains to a contract which has a forum-selection clause, the courts will enforce this unless special factors (such as unusual hardship, imbalance of power, etc.) are present.

Page 12: MBE Portion UBEBB

Interlocutory Orders An interlocutory order can take the form of a preliminary injunction or a temporary restraining order. An injunction is a court order telling someone to do or not to do something. It is an equitable remedy which means it seeks to do what is fair. A preliminary injunction preserves the status quo before final judgment, i.e. does not allow further harm to occur to the parties. The Plaintiff seeks the preliminary injunction and can only do so after a hearing takes place. To get a preliminary injunction, the court examines from the Plaintiff’s perspective:

• The likelihood of immediate and irreparable harm if an injunction is not issued, • The balance of the possible harm to the Plaintiff and Defendant if an injunction

is not issued (the harm that the Defendant would suffer if an injunction was granted must be lower than the harm that the Plaintiff would suffer if an injunction wasn’t granted),

• The likelihood of the Plaintiff’s success on the merits, and • Whether or not the injunction is in the public interest.

If an injunction is granted, the Plaintiff must post a bond which is then used to reimburse the Defendant for injury caused by the injunction if, at trial, the Defendant wins. Example (CivPro: Preliminary Injunction) After the hearing, Abby seeks an injunction against Charlie to prevent Charlie from reproducing more of Abby’s designs. The court would most likely grant Abby this injunction. If Charlie’s attorney disagreed, they would file for a motion to dismiss the injunction (covered later). The preliminary injunction is in place until the court reaches a deliberation on the case. Like a preliminary injunction acts as a court order preventing or enforcing certain behaviour, a temporary restraining order (TRO) manifests in virtually the same way. The differences lie in the procedural steps to obtain a TRO, which primarily does not have to be sought only after a hearing. A TRO is an emergency remedy used to maintain the status quo pending the outcome of a preliminary injunction application and, in determining whether to award one, the person moving for one must allege immediate and irreparable harm. This then promotes the court to evaluate the award of a TRO on the same bases as a preliminary injunction. Example (CivPro: Temporary Restraining Order) In order to prevent Charlie from quickly distributing and profiting off the stolen designs before a court rules on the case, Abby seeks a TRO from the federal district. The court awards her the order, which is in place until such a time that she can seek a preliminary injunction.

Page 13: MBE Portion UBEBB

Pleadings A pleading is a declaration made for a case and can take the form of:

• A complaint, • An answer, • A third-party complaint pursuant to Rule 14, • A crossclaim, • A counterclaim, • An answer to any of the above, and • A court ordered reply to an answer

The general pleading requirements are the details a Plaintiff needs to include in their complaint in a civil litigation. Rule 8 of the FRCP only impose three requirements for a Plaintiff to start a case:

• A short and plain statement of SMJ, • A short and plain statement showing that the Plaintiff is entitled to relief, and • The relief sought.

Example (CivPro: Pleading, Complaint) Abby decides to sue Charlie in State B and says in her complaint (the pleading) that the federal district has SMJ based on citizen diversity and the amount sought ($100,000 which is more than the $75,000 threshold). Abby also states that, because Charlie infringed on her IP, she was wronged. The wrong that Charlie committed entitles Abby to damages of $100,000. In response, the Defendant must:

• State in short and plain terms its defences to each claim made, and • Admit or deny the allegations asserted against it by an opposing party.

A denial must fairly respond to the substance of the allegation. In other words, it is not satisfactory for Charlie to deny any wrongdoing on the basis that Abby did not have an obvious copyright symbol on her work. The Defendant can give a general denial or a specific one. A general denial is where a party, in good faith, denies all the allegations of a pleading, including the jurisdictional grounds. If a party wishes to admit some and deny other allegations, they would enter a specific denial. Most denials in the UBE will be a specific denial. Thinking back to jurisdiction, venue, insufficient process and service of process, these all form the overall “bundle” of a pleading by the Plaintiff and, in the practical context, are considered allegations in and of themselves. Therefore, they must be denied or else the court down the line will generally consider this as an admission. If, however, a Defendant wishes to later un-waive these admissions, they can do so if:

• The application to un-waive was promptly made, and • It is prior to the hearing on the original motion.

A party may amend the pleading once as a right within the first 21 days after service of:

• The original pleading, or • A responsive pleading or pre-answer motion to the original pleading.

Page 14: MBE Portion UBEBB

This can be done without any permission from the court or mutual agreement but can only be done once. In all other cases, an amendment is allowed if:

• The opposing party consents, or • The court finds it just to do so. The court will generally allow it unless:

o Granting amendment would unfairly prejudice the other side, or o The amendment is frivolous, e.g. adding a second defendant with no

supporting facts. An amendment voids the previous pleading, meaning the prior one has no legal effect whatsoever. Example (CivPro: Pleading, Specific Defence) Abby’s claim is filed in the federal district of State B, which is proper venue. Her complaint alleges the same as above:

• Charlie stole her designs and profited off them, and • That act caused a loss of $100,000 to Abby

Charlie, upon receiving the complaint and summons, wishes to challenge it. He initially wants to deny everything, meaning he would make a general denial, but his lawyer advises him to tactically enter a specific denial. The specific denial admits that the jurisdiction, venue, process and service were all proper. In his pleading, Charlie denies that he profited off the designs since he wound up incurring a loss because of printing and other administrative costs. After tracing his steps, Charlie quickly remembers that the envelope containing the complaint and summons was placed on his desk at work. He investigates how it came there and discovers that it was a mailroom technician who left it there. Charlie contacts his lawyer and she advises him to file a motion to dismiss on the basis of improper service, and that he must do this quickly because it has been 15 days since the complaint and summons were served. There are some instances where a claim has already been filed but the harm of an incident is not fully realised until after the limitation has expired. In these instances, courts apply the relation back doctrine which enables a party to add to the claim so long as it arises out of the same transaction or occurrence as the original complaint allegations. The second arm of this doctrine concerns a Defendant, or rather, an additional Defendant. An additional Defendant can be related back if:

• They are an alleged cause of the same transaction or occurrence as the original claim,

• They received notice of the original action within 90 days of filing the original complaint,

• They knew that they should have been named as a party but wasn’t because of mistake or some other human error, and

• The original claim was filed in a timely manner. Example (CivPro: Relation Back Doctrine) It is 1 January 20XX, 3 years after Abby made the original complaint against Charlie. She comes to realise that Charlie not only stole her work, but also stole her own bespoke piece of software that she used to create that work. Therefore, the loss is

Page 15: MBE Portion UBEBB

much greater than $100,000 and is somewhere along the lines of $500,000. Under the relation back doctrine, Abby can seek to amend her claim even though the statute of limitations on copyright expires after 3 years. In her investigation, Abby discovers that Charlie used a computer hacker named Hank to bypass all the security features on her software, thus enabling Charlie to use it without a licence and profit from distributing it. Can Abby add Hank as another Defendant under the relation back doctrine?

No. 4 years is well beyond the time limits of 90 days set by the courts and, unfortunately for Abby, Hank gets off scot-free because her realisation came after the limitation period expired.

From here, there are numerous directions in which a proceeding can… proceed. The most prominent one is in the form of a counterclaim. A counterclaim is a claim against an opposing party and is broken down into two arms: compulsory counterclaim and permissive counterclaim. A compulsory counterclaim is one that must be made in the party’s pleading or else the party seeking it waives the right to it. For example, if Jack alleges that Tom committed nuisance by blaring his music too loud, Tom might claim that he did so to drown out Jack’s daily and nightly lawnmowing and thus Jack committed nuisance. This counterclaim must be made in Tom’s pleading and, for it to be valid, it must:

• Not require adding another party out of the court’s jurisdiction, and • Arise out of the same transaction or occurrence.

A permissive counterclaim is not a compulsory one. It sounds circular to say so but the underlying reason for this is to enable disputes between parties to be dealt with “in one go”. For example, still using Jack and Tom, let’s suggest that the only reason Tom was blaring his music out loud was because he was annoyed at Jack for punching him one day. This is not strictly related to the initial claim but its permission within the litigation will enable the overall animosity between the two parties to be resolved. Up until now, I’ve made a few references to the phrase “same transaction or occurrence” without giving a strong rubric as to how this is evaluated. In doing this, the courts will analyse whether:

• The issues of fact and law are largely the same, • Res judicata would bar a subsequent suit, • Substantially the same evidence supports or refutes the counterclaim, and • There is any logical relation between the claim and counterclaim.

I’ll discuss res judicata later. Let’s first look at another direction in which a suit can proceed: crossclaims. A crossclaim is where a party to a case may state a claim against a co-party but only if it arises out of the same transaction or occurrence. Example (CivPro: Crossclaim) Jack’s other neighbour, Kelly, agrees to join the suit against Tom. The docket is now Jack and Kelly v Tom, where Jack and Kelly are co-Plaintiffs. However, Kelly also

Page 16: MBE Portion UBEBB

agrees that with Tom that Jack’s incessant lawn-mowing was frustrating. Her feelings against Tom notwithstanding, Kelly can move to be a crossclaimant. The docket now reads: Jack (cross-defendant) and Kelly (cross-plaintiff) v Tom

Page 17: MBE Portion UBEBB

Joinder Some cases are a lot more complex than one Plaintiff suing one Defendant on one claim. Though they may start off as that, some cases can develop into a network of litigation. This has numerous poor effects, such as expense, time and stress, so the court enables what could be deemed as separate litigations to be joined into one. This is the essence of joinder. Joinder has two aspects to it: joinder of parties and joinder of claims. Joinder of parties is where, upon investigation, the number of parties on either side increases because of some factor. The FRCP rules are fairly straightforward with this; under Rule 19, joinder is mandatory if:

• A party is necessary to the case, and • Joinder is feasible.

There is also the permissive side to this, where a party may be joined if:

• The party asserts a right to joint and several relief, or the claim arises out of the same transaction or occurrence,

• A common question of law or fact exists, and • SMJ is present for each claim.

Basically, there are times when joinder is mandatory and other times when it is a “nice-to-do”. Joinder of claims is where there are claims which may arise that are not related to the original matter. Under Rule 18(a), a party can bring as many claims as it wants against the opposing side. This can take the form of an additional claim, a counterclaim, a crossclaim or a third-party claim (covered in implead). There are also instances where a non-party may wish to be a part of the case and join it as an intervener. They do not need the consent of the other parties and would exercise intervention as of right, governed by Rule 24(a) which states that a potential party has the right to intervene in an action if:

• They have an unconditional right to intervene by federal statute, or: o They claim an interest relating to the subject-matter of the action, o Their claim is not adequately protected by the existing parties, and o The disposal of the original claim may harm the non-party’s interest.

If intervention as of right is not present, the court still has discretion and may allow permissive intervention upon a timely motion pursuant to the requirements under Rule 24(b):

• They have a conditional right to intervene by federal statute, or • They have a claim or defence which shares with the main action a common

question of law or fact. If you’ve ever watched any legal show, you might be familiar with the term “class action”. But what is it? Let’s break it down. We know that an “action” is a cause of action, i.e. the claim. What exactly is a “class”? We have a broad idea that it generally means people with some commonality as to the claim. But you might ask, “Isn’t that just joinder?” And to that I say, “No, it’s not.”

Page 18: MBE Portion UBEBB

A class action is inherently distinct from joinder. In fact, a class action is used where joinder is impracticable. Let’s take a look at Rule 23(a), which outlines the requirements for a class action:

• Numerosity: the number of Plaintiffs must be so numerous that joinder is impracticable;

• Commonality: there are questions of law or fact common to the class; • Adequacy of Representation: the representatives will fairly and adequately

protect the class; and • Typicality: the claim and defence of representative parties are typical

If you want a mnemonic, you C(ommonality) A(dequacy of representation) N(umerosity) ' T(ypicality) joinder a class action: CAN’T. To certify a class, i.e. authorise the Claimants to proceed as one class rather than multiple individuals, Rule 23(b) states that it will be certified if:

• Separate actions would create a risk of inconsistent adjudications or harm the interests of other members,

• The grounds for relief are appropriate to the class, or • The questions of law and fact are predominantly the same and class action is

the best method to adjudicate the case.

Page 19: MBE Portion UBEBB

Discovery We’re finally here, the point where we get to use our investigative skills and do some real digging. Discovery is a concept that permeates every single CivPro curriculum because it is an important aspect in the pretrial process. Discovery is the phase in a civil litigation where each party clarifies the facts of the case by reviewing evidence. We’ll get onto evidence in a later section, for now though, the important thing is discussing what exactly is discoverable information. Under FRCP, a party may obtain all non-privileged information if:

• It is relevant to any party’s claim or defence, and • It is proportional to the needs of the case.

This information does not need to be admissible as evidence to be discoverable. So, what exactly does that mean? If you’ve ever done a logic test, imagine this scenario: All evidence comes from discoverable information but not all discoverable information becomes evidence. To initiate the discovery process, the Plaintiff must call a conference between the parties. This conference must be heled as soon as is practicable and at least 21 days before a scheduling conference. In this conference, a discovery plan is given from one side to the other and states issues such as the scope, the end-date of discovery and any other amendments to the deadlines for filing pleadings. The end-date of discovery is at least 60 days before the trial and the trial target date is usually 6 months to 2 years after the conference. Discoverable information is the precursor to what will be allowed as evidence, it is ensuring the parties have enough of an idea of the facts in order to help build up the overall parcel of evidence that furthers their argument. Under Rule 26(a)(1), without request each party must provide to the other:

• Contact information of individuals likely to have discoverable information, • A copy or description of all documents, electronically stored information, and

tangible things that the party may use to support its claims or defences, • Computation of each category of damages, and • Any insurance agreement that may be liable to satisfy the judgment or

indemnify the payments or reimbursements to satisfy the judgment. From this definition, what is considered as discoverable can take the form of documents, emails, databases, witness statements, depositions, communications, and pretty much everything that is relevant to the case. But there are limits to discovery. You have heard the term “attorney-client privilege” and thought that anything you tell a lawyer is confidential. That isn’t strictly true. The “privilege” is actually the “privilege to keep the information from discovery” but it has its limitations. Under Rule 26(b)(3)(a), litigation privilege is the protection of all materials prepared by an attorney (or agent) in anticipation of or during litigation unless:

• They are otherwise available under Rule 26(b)(1), and • There is a substantial need for it, and it cannot be obtained without undue

hardship. With that said, a party must disclose the existence of the material in sufficient detail to enable the other parties to assess the claim that it is protected by privilege.

Page 20: MBE Portion UBEBB

A deposition is a sworn, out-of-court, oral testimony of a witness that can be relied upon for the purposes of discovery. This is usually transcribed into a written statement. Under Rule 30(a)(1), a deposition can be taken without leave from the court except in the following instances, where it can be taken only with leave:

• If the deposition would result in more than 10 depositions being taken, • If the deponent has already been deposed, or • The deposition would last longer than 1 day of 7 hours.

If a party fails to provide adequate disclosure as outlined above, that party is not allowed to use that witness or information on a motion, hearing or trial unless the failure was substantially justified or harmless.

Page 21: MBE Portion UBEBB

Motions A motion is a request for the judge to make a legal ruling. This is not the same as a pleading and is usually after the hearing. A motion for discovery is one type, but other common ones include:

• Motion to dismiss (MTD), • Motion for summary judgment (MSJ), • Motion for judgment as a matter of law (JMOL), and • Motion for a new trial (MNT)

Motion to dismiss This is a request from either party to dismiss the suit as a whole, or in part, found in Rule 12(b). The standard for this requires the court to:

• Consider the facts in the light most favourable to the non-moving party, and • Determine if there is any basis upon which relief can be granted for the non-

moving party. Importantly, the court does not evaluate the merits of the case. In order for a motion to dismiss to be considered, it must be based on one of the following grounds:

1. A lack of SMJ, 2. A lack of PJ, 3. Improper venue, 4. Insufficient process, 5. Insufficient service of process, 6. Failure to state claim upon which relief can be granted, or 7. Failure to join a necessary party.

Challenging subject-matter can be done at any time, but grounds (2) to (5) are waived if they are not raised in the first responsive pleading.2 This is best illustrated by an example. Example (CivPro: Motion to Dismiss) Abby is still pursuing her former employee, Dani, for breaking the non-compete clause in their employment contract. She takes the case to a federal court district of State A since Dani resides broke the contract and State A. In her first responsive pleading, Dani would seek a motion to dismiss on the grounds of personal jurisdiction because the federal court does not have jurisdiction over these types of cases. Motion for summary judgment A summary judgment is a judgment that takes place before and instead of a trial. It too can be made on the whole or part of a case. The relevant rule is Rule 56 which outlines that the time to file a motion expires 30 days after the close of discovery unless altered by the court or local rules. In awarding summary judgment, the court grants one when:

• It considers the evidence in the light most favourable to the non-moving party, • There is no triable issue of material fact, and • The movant is entitled to judgment as a matter of law.

2 The notable exception is outlined in PAGE XXXX, where the court grants the challenge in some circumstances XXXXX.

Page 22: MBE Portion UBEBB

Effectively, the judge decides the case without the need for a jury trial because it is obvious who will win. Judgment as a matter of law This is governed by Rule 50. In JMOL, the judge rather than the jury will decide the case. This is done during the trial but before the case is submitted to the jury for deliberation. A motion for JMOL will be granted if:

• The non-moving party has been fully heard on the issue during the trial, and • The court finds that a reasonable jury would not have a legally sufficient basis

to rule in favour of the non-moving party on that issue. A party can renew its JMOL motion if it moved before the case was submitted to the jury:

1. Plaintiff makes case à Defendant makes JMOL motion.

2. Plaintiff makes case à Defendant presents defence à Plaintiff moves from JMOL.

3. Plaintiff makes case à Defendant presents defence à Defendant moves for JMOL.

4. Plaintiff or Defendant moves for JMOL à Movant is denied à Jury deliberates and makes verdict against movant à Movant motions for JMOL again (renewal).

The potential movant cannot raise the motion after the verdict if they did not raise it first. Motion for a new trial There may be instances where the evidence weighs heavily towards one side, but not heavily enough that JMOL would be successful. In these instances, under Rule 59, the moving party can file a motion for a new trial. There are other instances where a motion for a new trial is granted:

• Error at trial that makes its outcome unfair, • New evidence surfaced that could not be obtained by due diligence for the

original trial, • There was prejudicial misconduct, or • The verdict was excessive or inadequate.

In any of the scenarios above, including the verdict being against the weight of the evidence, the party seeking the new trial must file this no later than 28 days after the entry of the judgment.

Page 23: MBE Portion UBEBB

Sanctions A sanction is a punishment by the court for improper adherence to procedure. Honestly, there is no real way around learning this other than rote so here they are: 1. Failure to Preserve Electronically Stored Info (ESI) The sanction is found in Rule 37(e) and may be imposed when:

• ESI should have been preserved, • The party failed to take reasonable steps to preserve, and • ESI cannot be replaced or restored through additional discovery.

If the above test is satisfied, the court may:

• Order measures to cure any prejudice, or • If failure to preserve was intentional:

o Order an adverse inference instruction to presume or infer that the information lost was unfavourable to the offending party,

o Dismiss the whole action, or o Enter a default judgment.

2. Rule 11 Sanctions This title represents a category of sanctions imposed on attorneys and law firms that file frivolous documents with the court. Under Rule 11, all papers must be signed by an attorney and certify that:

• They are not being presented for any improper purpose, • That the legal contentions are warranted and non-frivolous, • That the factual contentions have evidentiary support, or will after discovery,

and • That the denials of factual contentions are warranted or reasonably based on

a lack of information. The court may issue a sanction, either by motion by a party or on its own, for the failure of adherence to Rule 11. The nature of these are at the discretion of the court and can be:

• A non-monetary directive (e.g. to rectify the papers), • A penalty (e.g. for wasting time), and/or • To pay reasonable attorney’s fees and/or expenses which resulted from the

violation of Rule 11. A party cannot file a Rule 11 motion with the court without first:

• Serving the motion on the offending party, and • Giving the opponent 21 days to withdraw the paper/pleading/rectify the mistake.

3. Pretrial Conference Sanctions These are sanctions relating to the failure to comply with the order for a pretrial conference and are found in Rule 16(f). The court can, through motion or by itself, impose any order if a party:

• Fails to appear at a scheduling or pretrial conference, • Is substantially unprepared to participate in the conference, • Does not participate in good faith in the conference, or • Fails to obey a scheduling or other pretrial order.

Page 24: MBE Portion UBEBB

The sanctions imposed may include: • A prohibition supporting or opposing designated claims or defences, • Striking the pleading, • Dismissing the action, or • Entering a default judgment.

Page 25: MBE Portion UBEBB

Claim and Issue Preclusion Preclusion is the act of one thing necessarily preventing the action of another thing. In legal terms, claim preclusion is known as res judicata. This is a bar on litigation that is applied to a case where the case has already been given a final judgment on the merits by a court of competent jurisdiction. The bar is so strong that it applies to any part of a case when the case has been given a final judgment. Res judicata may be invoked when:

• The parties are identical or in privity, which means that a non-party has a legally recognised relationship with an original party and would be bound by a judgment against the original party;

• Prior judgment was rendered by a court of competent jurisdiction; • The final judgment was made on the merits of the claim; and • The same claim is involved in both actions, i.e. the claim arises out of the same

transaction or occurrence. If the same claim is brought against the same party, the Defendant would respond by pleading res judicata. Example (CivPro: Res Judicata) Abby worked with Bill to create designs; these are ones that Charlie also stole. Abby has already had her judgment in the case against Charlie and won! Bill, on the other hand, is really annoyed at Charlie and wants to bring a suit of his own. Applying res judicata, Bill is in privity with Abby; prior judgment was rendered by a court of competent jurisdiction; the final judgment was made on the merits of the claim; and it is the same claim. Charlie would then respond in his pleading with res judicata. Another aspect to preclusion is issue preclusion or collateral estoppel. This is where, if a principle of issue or fact is decided in one case, the parties cannot relitigate that same issue or fact. It is another bar on litigation that prevents an issue from being retried if there has been a final judgment on the merits by a court of competent jurisdiction. Collateral estoppel may be invoked when:

• Valid and final judgment was issued in the prior action, • The issue is identical to the issue decided in the prior action, • The issue was actually and conclusively decided in the prior action, and • The party against whom enforcement is sought had a similar motive and

opportunity to defend the claim in the prior action.

Page 26: MBE Portion UBEBB

Jury Trials Under Rule 38, parties have a right to demand a jury trial. This is done by serving the other side with a written demand no later than 14 days after the last pleading directed to the issue is served; if this is not done in the specified timeframe, the right to demand a jury trial is deemed waived. The party requesting the jury trial must specify the issues it wishes to have tried by a jury, otherwise it is presumed that all issues in the case are to be dealt with at jury trial. Jury selection is the process of determining who comprises the jury. A jury cannot be fewer than 6 and no more than 12 people, with each person in the jury casting a vote. The result must be unanimous unless the parties to the case stipulate otherwise. Voir dire is the process by which juries are questioned about their backgrounds and, consequently, any biases they may have. If a juror has biases, they are struck meaning they can no longer serve on the jury. Example (CivPro: Jury Striking) On the list of potential jurors for the case regarding Abby’s designs is a juror named Chris. Chris and Abby used to work with one another before Abby left their company to start her own one. Because this situation could give rise to an unfair bias either way, Chris will be struck as a juror and no longer allowed to serve as a juror for this case. Jury instructions are a set of instructions given by the judge to the jury on how to determine the facts in the case which then ground the decision. They are often given in the form of, “If X fact is true, you must conclude Y,” which is significantly less complex than the previous standard language for jury instructions. At the close of evidence or at any time earlier that is reasonable under a court order, a party should file and supply the other parties the written requests for the jury instructions it wants the court to give. With that said, a party may also file requests for instructions after the close of evidence so long as the issues to which the instructions pertain could not have been reasonably anticipated at an earlier time. Obviously, with the leave of the court, a party can file requests for instructions outside of the time periods. If a party wishes to object, they must do so on the record and state the matter they object to and on what grounds. The grounds for objections are as follows: • The instruction is not an accurate statement of the law, • There are no facts in evidence that support giving the instruction, • The instruction is argumentative, • The instruction is cumulative, i.e. overbearing on a certain point, • The instruction conflicts with other instructions, • The instruction is confusing or unintelligible, or • The instruction is unnecessary.

Page 27: MBE Portion UBEBB

Appeals and Review An appeal is a last resort of sorts. It is made by the losing party whose attempts to dismiss, JMOL, etc. have all failed. To file an appeal, it must be done within 30 days of the entry of either:

• A final judgment, or • An interlocutory order.

These are strict unless an exception applies. A final judgment’s purpose is to avoid piecemeal litigation; in effect, this means that a ruling on the matter is done and there are no further issues to discuss. The judgment is final if:

• It resolves the claims in the action on the merits, and • It leaves nothing for the court to do but execute the judgment.

A federal appellate court can only hear an appeal from a final judgment on the merits unless an exception occurs. An interlocutory order is what I have defined as above. These are generally not appealable, however, there are some instances where they may be. Ultimately, to be appealed, they must be “unresolved” at the time of the final judgment. The most common type of interlocutory order subject to appeal would be an injunction, and in order to have grounds, the following must be satisfied:

• The order conclusively determines the disputed question, • The order resolves an important issue that is separate from the merits of the

action, and • The issues would effectively be unreviewable on appeal from a final judgment.

Once a district court enters an interlocutory order, the appellant has 10 days to seek an appeal. One exception to the general rule on appeals is the certified appeal exception, namely that an immediate appeal is allowed when:

• A federal district court judge certifies certain grounds for immediate appeal, and • The Court of Appeals agrees to permit the appeal.

Another exception is found in Rule 54(b), or the Rule 54(b) Exception. An immediate appeal is allowed when:

• The action has multiple parties or multiple claims, • The court directs final judgment for some of the parties or some of the claims,

and • The court expressly determines that there is no just reason for the delay of an

appeal. A review is different to an appeal. To understand them, let’s make a presumption that both will be “successful” and look at the effects. If an appeal is successful, it overwrites the previous conclusion on that matter. If a review is successful, it means that there was some procedural defect in the court below; it doesn’t necessarily guarantee a “win” over a specific issue. A party may seek to have a non-final order reviewed along with another appealable order currently pending before the appellate court when:

• The non-appealable and appealable decisions are inextricably intertwined, or

Page 28: MBE Portion UBEBB

• The review of the non-appealable order is important to the review of the appealable order.

Page 29: MBE Portion UBEBB

Constitutional Law Introduction The US Constitution (the Constitution) is a hotly debated topic in news and politics. From a legal perspective, it is the model by which all laws are created, developed and enforced. For the UBE, constitutional law focuses heavily on rights, freedoms, justiciability and judicial review. The section will deal with the following facts: Congress wants to install a new pneumatic tube system that is set to revolutionise interstate commerce. The system works by installing a network tubes with insertion points all around the different states on the continental US. Some states are against this because they think it will ruin the natural layout of the land.

Page 30: MBE Portion UBEBB

Separation of Powers In most modern democracies, the idea of a separation of powers is very clearly defined as the independence of power and makeup of the different arms of a state’s governance. Typically, these are:

i. The legislature (they make the laws), ii. The executive (they enforce the laws), and iii. The judiciary (they interpret the laws).

The UK’s legislature is Parliament, the ruling party of which forms the executive, and the judiciary is a separate entity. In the US, the legislative power is Congress, the executive power lies with the federal government (the leader of which is the President), and the judiciary power lies with the federal courts. These branches of governance have different powers and checks and balances to ensure no one branch conflates their power over the jurisdiction of another. CONGRESSIONAL POWERS This is not a guide on US politics, but some things are important to note. Congress is a bicameral legislature, which means it has two chambers:

• The House of Representatives, and • The Senate.

The House of Representatives is made up of 435 representatives, all of whom were directly elected. The number of representatives is determined by the state population. The Senate is made up of 100 senators, with each state having two. Representatives have a two-year term; senators have a six-year term. Congress has broad powers over:

• Taxes, • Spending, • Commerce, • War, • Investigations, • Properties of the United States, • Bankruptcy, • Citizenship, • Admiralty, • Coinage, • Patents/Copyright, • The “Necessary and Proper” power, i.e. the power to make all laws necessary

and proper for executing any of the above powers. Let’s deal with a few of those in greater detail. Starting with the Commerce Clause. Found in Article 1 §8 Cl.3 of the Constitution, the Commerce Clause gives Congress the power to regulate “commerce with foreign nations, and among the several states, and with the Indian”. What this means in effect is that Congress can regulate:

• People and instrumentalities of interstate commerce, meaning cars, planes, drivers, pilots, etc,

• Economic and commercial activity that has a substantial effect on interstate commerce, and

Page 31: MBE Portion UBEBB

• Channels of interstate commerce, meaning highways, internet connections, phone lines, etc.

Congress regulates all of these on a federal level, leaving intrastate regulations up to the individual states themselves. These federal regulations will be upheld:

• When there is a rational basis, • To conclude the cumulative impact, or • There is a substantial impact on interstate commerce.

The latter two of these will not be used when the activity itself is not commercial or economic in nature. Example (ConLaw: Commercial Clause) Congress wants to enact legislation that concerns the safety of a new pneumatic tube system that works across state lines to deliver goods in under 1 hour. Because this law concerns the instrumentalities of interstate commerce, this law is constitutional and thus is within the congressional power to legislate under the Commerce Clause. A state’s own legislature can regulate commerce if Congress has not enacted laws on the subject matter. If Congress has enacted legislation on that subject, any state or local law would be pre-empted, which means that Congress claims preclusion on the matter and thus supersedes it. A state cannot pass laws that:

• Discriminate against out-of-state commerce, or • Place an undue burden on interstate commerce.

Commerce laws are typically discriminatory if they are facially discriminatory or have a discriminatory impact. These are unconstitutional unless:

• The burden is narrowly tailored to achieve a legitimate, non-protectionist state objective and no less-discriminatory alternatives are available (think of proportionality), or

• The state is a market participant rather than a regulator of economic activity. Commerce laws are typically unduly burdensome and thus unconstitutional when:

• The burden on interstate commerce is, • Excessive to the proposed benefit to the state

Example (ConLaw: Unduly Burdensome) The pneumatic tubes are a go! The regulations for the pneumatic tubes state that each individual good cannot weigh more than 2000g and must be able to fit in a cylindrical tube with dimensions 200mm x 200mm x 1000mm. State B is generally opposed to the pneumatic tubes because of their subjectively unsightly appearance. Its main source of income is tourism to its luscious national parks, so they enact legislation which in effect prohibits private companies from accepting a tender that the federal government has out to construct these. This law would not be allowed because it places an undue burden on interstate commerce by increasing the out-of-state workload, thus being excessive in order to prevent the lack of tourism. The second of the congressional powers is the power to enforce the 13th, 14th and 15th amendments. Congress has the power to enforce:

• The 13th Amendment (the abolishment of slavery),

Page 32: MBE Portion UBEBB

• The 14th Amendment (privileges and immunities, due process, equal protection and the apportionment of representatives), and

• The 15th Amendment (that the right to vote cannot be denied because of race). The limit to this power is such that Congress can only prohibit behaviour likely to involve a constitutional violation. Congress cannot define constitutional rights or change the substantive law, and there must be congruence and proportionality between the injury to be prevented and the legislative means adopted. Example (ConLaw: Unconstitutional Legislation) State C wants to enact legislation that enables more than two people to be senators for the state. Congress can disallow this by deeming the legislation as unconstitutional. Another power of Congress is its power over taxation. Congress has the power to lay and collect taxes, duties, imposts and excises. Under the 16th amendment, Congress has the power to collect taxes on income from any source. These duties, etc. must be geographically uniform throughout the US. This means that these taxes must function with the same effect in every place, and not necessarily that the income from each state must be equal. In relation to taxation, Congress also has broad powers over spending, i.e. the allocation of resources collected from taxation. Congress has the power to spend for the common defence and general welfare. In addition, Congress can place restrictions or conditions on states receiving federal funds, but these must satisfy five conditions:

1. The spending must be for general welfare, 2. The condition imposed must be unambiguous, 3. The condition must be related to federal interest in national projects or

programs, 4. The condition cannot induce unconstitutional activity, and 5. The condition cannot be so coercive as to turn pressure into compulsion.

And, finally, the last of the relevant congressional powers is the power to delegate. In effect, this is Congress “passing off” legislation over areas that it otherwise does not care to legislate. A legislative power is delegable if:

• The Constitution allows it, and • Congress provides reasonably intelligible standards to guide legislation.

PRESIDENTIAL POWERS The President of the United States is the leader of the executive branch, or the head of the federal government. They have domestic powers and foreign powers. The domestic powers concern the execution of law, the appointment of officers, the removal of officers, the pardoning of federal crimes, and they are the commander in chief of the military. The President also shares some of its foreign powers with Congress, such as the implementation of treaties; these are negotiated by the President but must be ratified by the Senate. The President has the power to enter into Executive Agreements, which are agreements between the President and the head of a foreign country; and the power to control and deploy troops in foreign countries.

Page 33: MBE Portion UBEBB

JUDICIAL REVIEW This is the greatest power held by the judiciary and concerns the power that the court has to declare laws as unconstitutional. Judicial review is not a part of the Constitution, rather it evolved as a natural progression in determining the scope of the judiciary’s powers. At the crux of judicial review is the court’s ability to strike down actions, legislation and decisions made by governments on the basis that they are unconstitutional. This is used sparingly so as not to interfere with the 11th Amendment, which gives states sovereign immunity, basically broadly enabling them to enact any legislation they so wish to do and protecting them from lawsuits based on the enaction of those laws. Judicial review acts as a limitation to that. As will be discussed in later sections, in order to bring a case to the federal court and have them review it, the case must contain some essence of justiciability.

Page 34: MBE Portion UBEBB

Conflict of Laws This is a huge topic in the UBE. Because the US is comprised of two layers of legislation, federal and state, there can often be a conflict in how those laws are applied. There are multiple different theories on how to determine the choice of laws:

• Traditional Vested Rights, • Most Significant Relationship, and • Interest Analysis.

1. Traditional Vested Rights (TVR) Under this approach, if there is a conflict of laws, the court will apply the law of the state where the transaction or event occurred. 2. Most Significant Relationship (MSR) Under this approach, if there is a conflict of laws, the court will apply the law of the state having the most significant relationship to the transaction and the parties. 3. Interest Analysis (IA) Under this approach, the court weighs the interests of the states involved. The state with the greatest interest will have its law applied. In determining the greatest interest, there is a three-step process:

i. If one state has an interest the apply its policy and the other state doesn’t, the court should apply the law of the interested state;

ii. If there is an apparent conflict between the interests of two states, the court should apply the law of the interested state; and

iii. If there is still a conflict, the law of the forum state applies. In applying this to specific areas of law: TVR MSR IA

Torts Apply the state law where the wrong or injury occurred.

Apply the state law where the state has the most significant relationship to the occurrence and the parties. Courts must consider where the injury occurred, the domicile of the parties, and where the relationship of the parties is centred. If this is unclear, apply the local law.

Apply the law of the state with the greater interest.

Contracts w/o COL Provision

Apply the law of the state where the contract was formed or is to be performed.

Apply the law of the state has the most significant relationship. Courts must consider the place of contracting, the place of negotiation/performance, the location of the subject matter of the contract, and the

Apply the law of the state with the greater interest.

Page 35: MBE Portion UBEBB

domicile/residence/nationality of the parties.

Premarital Arrangements

Apply the law: i. Where the

agreement was executed, or

ii. Of the state that has the most significant relationship to the transaction and the parties.

Most states will use the MSR approach, where they will analyse:

i. Where the parties were married,

ii. Their residence while married,

iii. Where the parties are currently living,

iv. Where the assets are located, and

v. Where, if any, any children were born.

There are also different rules for contracts with a choice of laws provision, real property, and the inheritance of real and personal property. For a Contract w/COL Provision, the parties are free to choose for matters of contract construction. For contract validity, however, the parties may only choose applicable law if:

i. The state has some connection, ii. The contract was not entered into under fraud, duress or mistake, and iii. The choice of law is not contrary to a substantial policy interest of another

state that has more of a significant interest. Courts will also enforce a contractual forum-selection clause to transfer venue unless special factors are present (covered above as inequal bargaining power, etc.). Regarding property, the situs rule is a rule which states that the law of the state where the real property at issue is located. An important thing to note is the Restatement (Second) of Contracts (Rest. 2nd) which guides judges on evaluating this. Under Rest. 2nd, it is presumed that the situs state has the most significant relationship. If the land is merely incidental to a contract, e.g. a brokerage or a mortgage contract, the traditional choice of law rules apply. For the inheritance of real property, the situs rule is applied. If the inheritance of personal property, the law of the decedent’s domicile state is applied. The Law Applied… …by federal courts. The Erie Doctrine applies when a federal case is brought under diversity of citizenship jurisdiction and states that the federal courts will apply federal procedural law but must apply the substantive law of the forum state in which it sits.

Page 36: MBE Portion UBEBB

…by state courts. A state court will apply the law of the forum state for procedural issues. For substantive issues, the choice of law rules of the forum state determines which state’s law is applied. Procedural law relates to the civil procedure rules, limitations, the burden of proof, and rebuttable presumptions. Substantive law relates to the choice of law rules, frauds, irrebuttable presumptions, limitations that condition a substantive right or have a borrowing statute, and damages. Under the Supremacy Clause, federal law will always be supreme over conflicting state law. This is known as pre-emption and is divided into two arms: express and implied. Express pre-emption is where the law explicitly states that federal law is supreme, and implied pre-emption is fulfilled when:

• There is a direct conflict with state law, • There is field pre-emption, i.e. where Congress never explicitly states that a law

is pre-empted, but regulates in that “field” of law so much so that it amounts to it, or

• The state law interferes substantially with the objective of the federal law. Under the 10th Amendment, all powers not granted to the federal government are reserved to the states. Whilst Congress cannot compel a state government to implement legislation, it may induce the state government to do so by attaching restrictions and conditions on federal funding. Example (ConLaw: The Law Applied by Federal Courts) Sam, a resident of State G, was driving in State F when they collided with James, a resident of State H. James brings a negligence claim of $100,000 in damages against Sam in a federal district court of State H. The court will apply the federal procedural law to ensure jurisdiction, venue, etc. It will also apply the substantive law of State H because that is where the court sits. The full faith and credit clause is found in Article IV §1 of the Constitution and imposes an obligation on states to respect the “public acts, records and judicial proceedings of every other state”. A judgment is entitled to full faith and credit when:

• The court had jurisdiction (personal and subject-matter), • The case was decided on the merits, and • The judgment was final.

The effect of this is that state courts are required to give a federal judgment the same force and effect as it would have been given under the preclusion rules of the state in which the federal court sits so long as the law does not violate the state’s public policy. Example (ConLaw: Full Faith and Credit Clause) The federal district court in State X has recently issued a final judgment on a matter of law pertaining to the rescission of a tender to complete construction of the pneumatic tube system. The ruling was that states’ governments can, at will, rescind the offer. Building Co., incorporated and located only in State Y, tries to sue its local government in state court for rescinding the tender offer. Under the full faith and credit clause, the

Page 37: MBE Portion UBEBB

judgment in State X was done with jurisdiction, decided on its merits and final, so the claim that Building Co. has against State Y will move no further under preclusion. There are also some instances relating to domestic disputes where the law applied varies from what is above:

Instance Rule Ex parte divorce, where only one spouse is before the court

This may be maintained without personal jurisdiction over the absent spouse if the Plaintiff spouse is a domiciliary of the court where the state sits

Divorce (non ex parte) If it is validly granted in another state, it is entitled to full faith and credit in all other states

Matrimonial action involving economic or child custody/support issue

In order to adjudicate, the court must have personal jurisdiction over the Defendant spouse

Ceremonial marriage The validity of a marriage is determined by the law of the state with the most significant relationship to the spouses.

Common law marriage A marriage that is valid when it was formed is generally valid everywhere unless it violates the strong public policy of another state. Most states will honour a common law marriage validly established in another state.

Page 38: MBE Portion UBEBB

Justiciability So far, we have discussed cases in the abstract with some pertinent examples used to illustrate the concepts. Now, however, we need to identify what comes before all of this. The idea of justiciability is one that relates to two overarching concepts:

• Standing, and • Real case or controversy.

Standing is the procedural grounds on which a potential Plaintiff stands in order to bring a case to court. This is different to an initial pleading, where the Plaintiff outlines the redressability of the issue in relation to damages. Standing exists when:

• The Plaintiff personally suffered injury in fact, • There is causation, and • The injury is redressable by court order.

For a Plaintiff to suffer injury in fact, they must either be injured or show that injury is imminent. If the Plaintiff is seeking injunctive relief, they must show an above average likelihood of future harm. A third-party may also have standing in lieu of a would-be Plaintiff if they can show either of the following:

• They have a close relationship with the would-be Plaintiff, • It is difficult or unlikely that the would-be Plaintiff would assert their rights on

their own, or • The would-be Plaintiff is an organisation.

For an organisation to have standing, they must show:

• That the case relates to an issue germane to the organisation’s purpose, • That the members would have standing to sue, and • That the participation of the members is not necessary.

Real case or controversy is basically saying that the court will not give advisory opinions or address hypothetical disputes. The yardstick for determining if there is a real case or controversy is limited by: ripeness and mootness.

Ripeness is whether the case is ready to be litigated. The court may grant a pre-enforcement review after considering the hardship that would occur if no review was granted and the fitness of the record to do so. Mootness is whether the dispute has ended or was resolved before review. If the case is moot, it will not be reviewed. There are some exceptions to when a case can be reviewed even if moot:

• Voluntary cessation, i.e. the parties withdrew the case, but it can resume at any time, or

• Class actions, where one more members has an ongoing injury The incorporation doctrine states that most Amendments to the Constitution are applicable to the states through the 14th Amendment, the Due Process Clause. The exceptions to this are:

• 3rd Amendment,

Page 39: MBE Portion UBEBB

• 5th Amendment, • 7th Amendment, and • 8th Amendment.

In general, the conduct of private actors in not protected by the Constitution. Courts will find government action of a private actor when it is a traditional public function or significant government involvement exists to authorise, encourage or facilitate unconstitutional conduct.

Page 40: MBE Portion UBEBB

Individual Rights Individual rights are those outlined in the Constitution as both rights and freedoms. A right is what the individual gets from the nation, and a freedom is what the individual gives to the nation. For example, I have a right to privacy, and I also have the freedom to express myself. You may see rights and freedoms get conflated in the phrase, “I have right to freedom of X,” and what that really means is that the nation gives you a right to exercise the freedom. Due Process Due process is the idea that no person shall be denied life, liberty or property without due process of law. There are two arms to this: procedural and substantive. Procedural due process is the process required when the government seeks to deprive an individual of such rights. To determine the procedure, the court applies the test from Matthews v Eldridge:

• The importance of private interest, • The risk of error under the current procedure and the value of additional

procedures, and • State interests and the burden that arises from additional safeguards.

In effect, the court balances these and seeks to evaluate the extent to which due process should be applied considering the case at hand. Substantive due process is an issue when the government regulates a Constitutional right. In determining whether substantive due process is an issue, two tests are applied:

1. Strict Scrutiny, or 2. Rational Basis.

1. Strict Scrutiny This is a test applied when the subject-matter is a fundamental right, i.e. Privacy rights, the right to vote, interstate travel, 1st Amendment rights, or the right to refuse medical treatment. The test is that the government has to show that the law is necessary to achieve a compelling government interest (proportionality). In effect, the law must be no more than is necessary to achieve the outcome. 2. Rational Basis When regulating a non-fundamental right, the law must meet the rational basis test, i.e. the Plaintiff must show that the law is not rationally related to a legitimate government interest. Equal Protection Equal protection is outlined in the 14th Amendment and it prohibits the government from denying equal protection of its laws. A law is discriminatory if:

• It is prima facie discriminatory, • It is facially neutral but is applied in a discriminatory way, or • A discriminatory motive for the law exists.

In order to assess whether a law is discriminatory, it must also have an effect on a class of people. These hypothetical classes are divided into suspect, quasi-suspect

Page 41: MBE Portion UBEBB

and all other classes. If a group affected by a Constitutional decision belong to one of these, it suggests that they are likely the subject of discrimination. In order to analyse whether equal protection has been violated, there are different tests for the different classes:

Class Comprised Test Applied Suspect Race, nationality, alienage

or infringement on a fundamental right

Strict scrutiny: the government must show that the classification is necessary to serve a compelling government interest

Quasi-suspect Gender, non-marital child, undocumented immigrant

Immediate scrutiny: the government must show the classification is substantially related to an important government interest

All other -- Rational basis: the Plaintiff must show that the classification is not related to a legitimate government interest

The remediation of past discrimination is allowed if the class actually suffered past discrimination. Example (ConLaw: Discriminatory Legislation) State D is frustrated with the number of people from the United Kingdom who migrate there because of its strong British influence, so it passes a law which prohibits anybody from the United Kingdom marrying a resident of State D. Because this both infringes on a fundamental right (marriage) and is a law concerning nationality, it is subject to strict scrutiny. Takings Under the 5th Amendment, the government may take private property for public use if it provides just compensation. It is deemed public property so long as there is a reasonable belief that it will benefit the public. Just compensation is determined as the “fair market value at the time of taking”. If the property is taken before compensation is paid, interest accrues from the time of taking. The concept of government takings is better understood in a negative definition, i.e.: the government cannot take private property for public use without just compensation. There are two types of taking: possessory and regulatory. Possessory taking is where the government physically occupies the property, even if it is just a small portion; it is also a per se action meaning there is no need to show that the act of taking has rendered the property economically unviable. Regulatory taking is where the economic viability (enjoyment) of the property is rendered

Page 42: MBE Portion UBEBB

untenable due to regulation by the government. The courts analyse whether regulatory taking has occurred by using the Penn Central test:

• The economic impact of the regulation on the Plaintiff, • The extent of interference with investment-backed expectation, and • The character of the regulation.

Example (Possessory Taking) Construction for the pneumatic tubes is well underway and is rolling out in a phased approach. In Phase 2 of the rollout, the construction is set to break ground in State F, a place with limited federal land in the form of national parks. The federal government wants to use some of the farmland, of which State F has plenty, to ensure minimal disruption to the busy metropolitan areas. They would need to provide just compensation to the property owners. Privileges and Immunity Clause Found in Article IV §2 of the Constitution, the Privileges and Immunity Clause says that states cannot intentionally discriminate against non-residents concerning:

• Civil liberties, or • Important economic activities.

With that said, regulations made by states will be upheld if the state can show: • A substantial reason for the difference, and • The law has a substantial relationship to the reason.

Example (ConLaw: Privileges and Immunity Clause) State B, still deftly against the pneumatic tubes, enacts legislation that prohibits any out of state company from accepting a federal tender for construction of the tubes if the works are to be carried out wholly or in part within the boundary of State B. Is this allowed? Well, while the rule is that the P&I Clause does not allow states to intentionally discriminate against non-residents concerning civil liberties or important economic activities, and indeed the construction of the pneumatic tube is a sufficiently important economic activity, the P&I Clause does not protect immigrants and corporations, meaning that an out-of-state construction company cannot claim discrimination. Furthermore, the regulations imposed by State B satisfy the reason for being upheld on the basis that there is a substantial reason for the difference and the law itself has a relationship to the reason. Other Freedoms The Constitution confers other rights, protections and freedoms to its subjects. These are plentiful. Within the 1st Amendment, they are:

• Protection of: o Speech/expressive activities, o Exercise of religion, o Press, o Peaceable assembly, o The right to petition;

• Congress also should make no law concerning the establishment of religion.

Page 43: MBE Portion UBEBB

Freedom… Breaching Regulation

Allowable Regulation

Other Details

Of speech Unduly Vague: does not put the public on reasonable notice as to what is prohibited Overbroad: regulates more speech than is constitutionally permitted

--- ---

Of speech (prior restraint)

--- Prior Restraint: the attempt to prohibit speech before it happens through a licensing requirement or court order. These are generally disfavoured and unconstitutional unless national security is at stake

Must satisfy strict scrutiny. Procedural safeguards for licensing are permitted if:

1. The government has an important reason,

2. There are specific, articulated standards to remove the discretion, and

3. Procedural safeguards are in place, including a prompt final judicial decision when a licence is denied.

Of speech (content-based restrictions)

--- Content-based restrictions: a subject matter or viewpoint is subject to strict scrutiny.

---

Page 44: MBE Portion UBEBB

Of speech (content-neutral restrictions)

--- Time, place and manner restrictions are allowed

Allowed if: 1. They are

narrowly tailored to achieve a significant government interest, and

2. They leave open alternative channels of communication.

Public forums --- --- Traditionally available to the public for speech. Regulations may include impositions on time, place and manner of content-neutral speech. If this is a content-based regulation, it must satisfy strict scrutiny.

Designated/Limited public forums

--- --- Not traditionally made available to public but the government may choose to make these available. If made available, they are subject to the same regulatory restrictions as imposed on public forums.

Non-public forum --- Traditionally limited. The government may regulate speech if:

1. It is reasonable to do so, and

2. The viewpoint discussed is neutral.

---

Private property There is no right to access another’s private property.

--- ---

Page 45: MBE Portion UBEBB

Of the press --- --- This freedom protects the press from liability when publishing:

1. Truthful private facts,

2. Regarding a matter of public concern, where

3. The information was legally obtained.

Commercial speech

--- The government may regulate truthful, non-misleading commercial speech if:

1. It directly advances a substantial governmental interest, and

2. The regulation is no more extensive than necessary to serve that interest (proportionality).

False or misleading commercial is not protected by the Constitution.

Symbolic speech --- Regulation if: 1. The

government has an important purpose which is,

2. Independent of the suppression of speech, and

3. The restriction is proportionate.

---

Fighting words and true threats

--- --- These are not protected. These are words which by their very utterance:

1. Inflict injury, or 2. Tend to incite an

immediate

Page 46: MBE Portion UBEBB

breach of the peace

Obscenity --- --- These are not protected. Material is obscene if it:

1. Appeals to “prurient interest”,

2. Is patently offensive, and

3. It lacks any redeeming artistic, literary or scientific value.

Association --- The government may punish a person’s membership if:

1. The group is actively engaged in illegal activities,

2. The member knows of the group’s illegal activities, and

3. The member has the specific intent of furthering such activities.

This is a fundamental right.

Establishment --- Laws do not discriminate if:

1. The law has a secular purpose,

2. The law’s primary effect neither advances nor inhibits religion, and

3. The law does not excessively entangle government with religion.

Any laws that discriminate against a religion are subject to strict scrutiny.

Page 47: MBE Portion UBEBB

Exercise of religion (Free Exercise Clause)

--- Laws of general applicability do not offend the Free Exercise Clause.

Any laws that interfere with a religion are subject to strict scrutiny.

Page 48: MBE Portion UBEBB

Dormant Commerce Clause Referring to the section on Congressional Powers, the Dormant Commerce Clause is an extension of the ordinary Commerce Clause and is an inference of the codified clause. In effect, it is said that, when Congress remains dormant on an issue regarding commerce, states cannot pass regulations that are either discriminatory or unduly burdensome against other states, thus affecting interstate commerce, without good cause. Discriminatory laws are evaluated on the basis of strict scrutiny and are almost always struck down, and unduly burdensome ones are evaluated on a burden-benefit analysis which asks the assessor to weigh the effects of the law on interstate commerce against the state’s interest served by the law. Example (ConLaw: Dormant Commerce Clause) Back on the pneumatic tubes, the new federal guidelines have outlined that the tubes must be placed at a minimum of 100ft above ground-level and have no fewer than 10 parcels within a 500m distance of one another. State B’s opposition to the pneumatic tubes has caught on. In particular, by State V, who proposes legislation that effectively means all parcels with a total volume of less than the limit for the pneumatic tubes must pass through the state via the state’s local postal delivery companies. This is an unduly burdensome law and its effects must be weighed against the state’s interest served by the law.

Page 49: MBE Portion UBEBB

Contracts and Sales Introduction Contract Law is the law which governs agreements made between legal persons within the confines of a regulatory framework. Whilst the basics of Contract Law are fairly congruent across jurisdictions, the US distinguishes contracts pertaining to the Uniform Commercial Code (UCC) and others. Uniform Commercial Code (UCC) vs Common Law Article II of the UCC states that the UCC applies to sales of goods contracts. A good is something that is movable at the time of identification, meaning most things that can be considered as personal property. Importantly, real property is not covered by the UCC. The common law applies to all other contracts, however, there are instances where the substance of a contract is mixed, in which case the predominant purpose of the contract determines which law (UCC or common) governs.

Page 50: MBE Portion UBEBB

Formation, Modification and Enforceability For a contract to be validly formed, it must possess:

1. Mutual assent, 2. Consideration, and 3. No defences to enforceability.

1. Mutual Assent Also known as offer and acceptance, mutual assent is the idea that the parties in the contract have exchanged some concurrence as to the intention to enter into a contract. An offer is defined as the manifestation of intent to enter into a contract, with definite and specific terms, communicated to an identified offeree. An offer can be terminated before it is accepted by either:

a. Rejection or counteroffer, b. Lapse, c. Revocation, or d. Death or Incapacity.

Termination is different to revocation insofar as revocation is a form of termination. An offer is deemed revoked through unambiguous words or conduct indicating unwillingness or inability to contract and can be direct or indirect. Indirect revocation can be when:

• The offeror takes definite action inconsistent with entering into a proposed contract, and

• The offeree acquires reliable information to that effect. Example (Contracts: Offer and Revocation) Samwise possesses 20 rings and makes an offer to Frodo to purchase them for $70. Before Frodo can accept the offer, Gimli approaches Samwise, takes him to the side, and enters into a contract to buy the rings for $100. Frodo hears this. The offer made from Samwise to Frodo is thereby indirectly revoked. There are some offers which are irrevocable, and these are:

• Option contracts, where the offeror gives the offeree an option to buy or sell a particular asset at a later date,

• Merchant’s firm offer, where the merchant offeror makes an offer to buy or sell goods made by a merchant. This offer is firm and irrevocable if it is in writing and signed by the offeror, or

• Offers made under detrimental reliance, i.e. the offer made by the offeror to the offeree was relied on by the offeree to the offeree’s detriment.

For an offer to be a validly constructed one, it should not have indefiniteness, which is understood as unable to be ascertained to a reasonable degree of certainty. If a contract is indefinite to duration, the contract is invalid. Another thing to note is the idea of an open price term, which does not necessarily make an offer invalid if both parties are merchants because the only essential term is quantity. If the contract is mute on the price, the UCC supplies a reasonable term regarding price. On the other side of mutual assent is an acceptance, which is a manifestation of assent to the terms of the offer. Acceptance can manifest in the act of performance. Specifically, for a bilateral contract, the start of performance constitutes an

Page 51: MBE Portion UBEBB

acceptance. For a unilateral contract, the start of performance only makes the offer irrevocable and acceptance manifests when performance is completed. A bilateral contract is a contract where both parties exchange promises to perform; a unilateral contract is a contract where the offeror promises to pay after the occurrence of a specified act. Example (Contracts: Bilateral Contracts) Most types of contracts, e.g. buying something at a shop, engaging in business, etc. Example (Unilateral: Unilateral Contracts) Frodo has lost his dog, Gollum, and places missing posters all over the Shire offering the person who finds the dog and returns it to him a reward of $1,000. This is an example of a unilateral contract because the contract is formed openly from Frodo, the offeror, to anyone who can perform the specified act (finding the dog). An offer can be rejected, which is defined as the manifestation of intent not to accept an offer. This terminates the offer when it is communicated, meaning that the offeree cannot accept the offer once it is rejected. If, on the other hand, the offeree responds to the offer with a counteroffer, this has the effect of both rejection and is a new offer from the opposite side. A contract implied-in-fact is a contract that is created by conduct if that conduct is intentional and each party has reason to know that the other party will interpret the conduct as an agreement. Example (Contracts: Implied-in-Fact Contracts) Ian and Elijah have engaged in various businesses together, their next project is a tell-all book about their experiences working on a three-part epic movie series. Ian says that he will write every other chapter and the foreword, while Elijah says he will do the chapters that Ian doesn’t do and the acknowledgements. They agree to split the profits from the book 50:50. They both complete their various parts and the book sells well, taking in over $100,000 in just the first 3 hours of sale. Though there was no contract between the two, it was implied in fact because they both held up their sides of the agreement. The mailbox rule (also known as the ‘postal rule’ in other jurisdictions), is the doctrine that state that acceptance sent by mail is deemed to be accepted once placed in the mail. Revocation sent by mail is deemed effective when it is received by the offeree. Under the common law, there is the idea of a mirror image rule, which states that the acceptance must exactly mirror the offer. If this is not the case, any variations constitute a counteroffer. However, if a contract is made under the UCC, the mirror image rule does not apply as long as:

• Both parties are merchants, • The change is not material, i.e. likely to cause hardship or surprise, • The offer does not expressly limit acceptance to the exact offer, and • No objection was made within a reasonable time.

2. Consideration This is the meat of the contract. Consideration is defined as the substance bargained for exchange of any act or forbearance that benefits the promisor or causes detriment to the promisee. Consideration must be made at the time stipulated in the contract.

Page 52: MBE Portion UBEBB

In other words, past consideration is not sufficient. There is a slight “exception” to this idea called the material benefit rule; this states that a promise made in recognition of a benefit previously received is binding to the extent necessary to prevent injustice, but this is not applied when the previously received benefit was conferred as either a gift or the enforcement of this as valid consideration would be disproportionate to the benefit conferred. Exchange of consideration can be imputed through the doctrine of promissory estoppel, which is better understood as a “shield, not a sword”. Promissory estoppel applies when:

• A party reasonably and foreseeably relied to their detriment on the other party’s promise (detrimental reliance),

• The promisor reasonably expected a change in position in reliance of the promise, and

• Enforcement is necessary to avoid injustice. As a result, promissory estoppel is not so much a method of enforceability as much as it is a method against unenforceability. Detrimental reliance is the idea that the person claiming it suffered some detriment as a result of reliance on the promisor’s promise. Modification There may be instances when the terms of a contract need to be modified but the contract has already been entered. Though past consideration is generally insufficient under the pre-existing duty rule, it may become sufficient if there is an addition or change in the performance of the contract, or there is an unforeseen difficulty that is so severe it excuses performance. Example (Contracts: Pre-Existing Duty) Legolas and Arwen hire a contractor, Pippin, to complete work on their holiday home and decide that they want a pool, a home theatre and a stable. Pippin observes the scope and decides to do it for the agreed price of $40,000. Near the end of the agreed time, there is an invasion by the Orc army which completely destroys the work that Pippin has done. Because this was an unforeseen difficulty, the prior performance of the contract is excused meaning the contract can be modified. Another example of modification comes under the UCC exception and concerns UCC contracts. This states that any agreement modifying a contract needs no consideration to be binding if the modification is done in good faith, however, it must be in writing if:

• The contract falls under the Statute of Frauds, or • The original contract states that modifications must be made in writing.

3. Defences to enforceability There are some instances which preclude a contract from being validly formed, therefore, rendering the contract unenforceable, outlined in the table below:

Issue Requirements Effect Example Incapacity The party does not

possess capacity, e.g. a minor, someone with

Voidable by the person

Entertainment Co LLC hosts a competition

Page 53: MBE Portion UBEBB

some specific mental illnesses, etc.

who lacked capacity

where participants who correctly identify the 4 movies released by them in 2019 wins $1,000. B, a minor, calls in and gets the answer right. B cannot enforce the contract because B is a minor.

Economic duress 1. Improper or wrongful threat made which,

2. Induced a party to enter into a contract, and

3. The threatened party had no reasonable alternative to enter into a contract.

Voidable G and K have an ordinary contract and, when it is up for renewal, G says to K, “I’m the only supplier of the Goods so I’m going to hike prices up substantially if you don’t increase your performance.” This is economic duress and is unenforceable.

Mutual mistake 1. Both parties are mistaken as to a basic assumption,

2. The mistake is material, and

3. The person asserting the mistake did not bear the risk of the mistake

Rescission or reformation

C and D form a contract. C is under the impression that he is selling 5 oranges for $2, whereas D is under the impression that he is buying 7 apples for $1.50. This is an example of mutual mistake and the contract is either rescinded or reformed.

Page 54: MBE Portion UBEBB

Unilateral mistake --- Voidable by mistaken party

The contract between C and D is for C to sell D 8 apples for $1.80. D, however, thinks it is to buy 8 apples for $1.65. This is a unilateral mistake and is voidable by D.

Misrepresentation --- --- --- Non-fraudulent misrepresentation

1. A statement of material fact made by,

2. A party or an agent, which is

3. False to the extent that it

4. Induces the contract, and

5. The other party reasonably relies on the misrepresentation to their detriment

Voidable by the injured party

C is growing kiwi fruit in their garden and advertises these as “beautifully tart, sweet and full of good nutrients” on January 1 2020. C sells out of the pre-orders, with one person saying that they can’t wait to eat all the good nutrients. On January 5 2020, a study comes out which conclusively labels kiwi fruit as the “most unhealthy fruit to ever exist”. This would be an example of a voidable, non-fraudulent misrepresentation because C did not know that kiwi would later be labelled as such.

Fraudulent misrepresentation

A party: 1. Knowingly, 2. Makes a false

representation, of 3. Material fact, and

Voidable by the injured party

Page 55: MBE Portion UBEBB

4. The other party reasonably relies on the misrepresentation to their detriment

Illegality Courts will not enforce contracts that are illegal or contrary to public policy.

The contract is void if the act is illegal at the time of execution. Performance is discharged if the contract subsequently becomes illegal. A contract with an illegal purpose is voidable by the party who did not know that the purpose was illegal.

Unconscionability It “shocks the conscious” of the court

Voidable by weaker party

Procedural One party has superior bargaining power over the other and uses that to their advantage

---

Substantive The contract contains terms that are obviously unfair and one-sided in favour of the party with the greater bargaining power

---

Frustration occurs when:

• Both parties knew the purpose of the contract at the time of formation, but • An unforeseeable superseding event outside of the parties’ control occurs,

which • Destroyed the purpose or value of the contract.

The Statute of Frauds is when a contract must be in writing in order to be enforceable. Not all contracts fall under the SoF but, those that do, must:

• Be in writing, • Be signed,

Page 56: MBE Portion UBEBB

• Identify and describe the subject matter, and • State all the essential terms.

In order for a contract to be subject to the SoF, it must fall within the following categories, remembered by the mnemonic: MY LEGS:

• Marriage, • Year, i.e. the contract cannot be fully performed within 1 year, • Land, i.e. the contract pertains to the sale or the creation of an interest in real

property (land), • Estate, i.e. the execution of a decedent’s estate, • Goods3, i.e. the sale of goods for $500 or more, and • Suretyship, i.e. the promise to answer or pay for the obligations of others unless

the main purpose of the promise was to benefit the promisor. Example (Contracts: Statute of Frauds) Bilbo contracts with Frodo to sell his house in the Shire for $150,000. Because the contract concerns the sale of real property, it must be in writing, be signed by both parties, correctly identify the and describe the house, and state the essential terms such as price, what is included in the price, and any annexations to the property.

3 There are exceptions to the Goods element:

- Merchant’s confirmatory memorandum which is a contract between two merchants that only needs a writing signed by the party enforcing it,

- Goods accepted or paid for, where only the goods already dealt with are exempt but not the whole contract,

- Custom made goods, where there has been a substantial start and not for sale in the ordinary course of the seller’s business, and

- An admission during a judicial proceeding

Page 57: MBE Portion UBEBB

Interpretation Interpretation concerns the role of the courts within the contractual dispute. Most disputes are generally dealt with through negotiation, but from the litigious, the court must follow various guidelines to then apply over the interpretation of a contract. Focusing on some in particular, let’s first look at the parol evidence rule which states that a party cannot introduce a prior or contemporaneous agreement that contradicts a later writing. This “parol evidence” is given orally or in writing. As always, there are exceptions to this rule:

• Parol evidence is allowed to correct a clerical error or typo, • Parol evidence is allowed to establish a defence against the validity of the

formation of the contract, • Parol evidence is allowed to interpret vague or ambiguous terms, however

courts interpret the words of the contract using the plain meaning rule, or • Parol evidence is allowed to supplement a partially integrated writing.

A partially integrated writing is writing that does not contain a complete statement of all the terms of the contract, and parol evidence is allowed so long as it supplements the writing rather than contradicting it. This is different to fully integrated writing, which is a complete and exclusive statement of terms that generally discharges prior agreements. Example (Contracts: Parol Evidence, Correction of Typo) Bilbo writes out the contract which states: xx. “… the sale of my cottage, Bageend … and some of the annexes for the price of $150,000”. Frodo and Bilbo both sign this. This is a contract in which parol evidence would be allowed, because there is an error in the name (it should be Bag End, not Bageend) and to clarify what annexes are included in the price of $150,000.

Page 58: MBE Portion UBEBB

Performance, Breach and Discharge Performance is the act of doing the action that a party is contracted to do, i.e. performing their obligations. If a party fails to perform their obligation, they are in breach of contract. In a more complex contract, there is the idea of divisible performance which basically means that the failure to perform one aspect of a contract does not necessarily put the promisor of performance in breach of the whole contract. In effect, the law treats these different and multiple obligations as separate contracts, so performance of each part entitles the party to payment for that part. In the event that there are express conditions precedent, i.e. express conditions which must be met before the other party performs their obligations, e.g. “Payment must be made before delivery of the Goods”, these are excused by either:

• The protected party’s failure to co-operate, or • Waiver, where the protected party voluntarily gives up protection (though this

can be retracted if the other party has not relied on it). Every contract has an obligation of good faith and fair dealing, i.e. there is an assumption that the parties are acting fairly and honestly. UCC contracts require honesty in fact, i.e. a merchant’s duty is imputed as honesty in fact and the observance of reasonable commercial standards of fair dealing in the trade. If a party does not render substantial performance under a common law contract, they have committed a material breach if the court decides so upon analysis of:

• The extent of their performance, • The hardship caused by failure to complete performance, • The adequacy of compensation for loss to the aggrieved party, • The likelihood that the breaching party will cure their actions (perform them),

and • Whether the breach was intentional.

A material breach from one party excuses the non-breaching party’s performance. On the other side, if the breach is deemed to be a minor breach, this does not excuse performance by the non-breaching party however they may bring a separate action for damages resulting from the minor breach. For UCC contracts, there is the idea of a perfect tender, where:

• The seller must deliver the concerned, conforming goods, but the seller must bear in mind that

• The smallest non-conformity is a breach and the buyer may reject all or a portion of the goods, but, if they choose to do so, then

• A rejection of goods must be made within a reasonable time. However, there are some exceptions. For example; the seller has a right to cure which enables them to remedy their non-compliance by taking steps to ensure compliance or otherwise cure the breach. This arises in either of the following scenarios:

• The time for performance (outlined in the contract) has not expired, or • The reasonable time to perform has been extended to substitute goods that

would be accepted. Another exception is if the contract concerned is an instalment one. An instalment contract is a contract which requires or authorises the delivery of goods in separate

Page 59: MBE Portion UBEBB

lots to separately accepted. A buyer can reject any instalment which is non-conforming (to the terms of the contract) if the non-conformity is to the extent that it substantially impairs the value of the goods. In other words:

• The performance may only be cancelled where an instalment is so defective that it substantially impairs the value of the entire contract, then

• The buyer can reject an instalment only if non-conformity substantially impairs that instalment and the time to cure has passed.

Under UCC contracts, if a buyer fails to reject goods after the reasonable opportunity to inspect the goods, the goods are deemed accepted. After the acceptance, the buyer can only revoke acceptance if either:

• The non-conformity of the goods substantially impairs their value, or • The defect was latent (difficult to discover), or

o The buyer had a reasonable assumption that the defect would be cured. The revocation of acceptance of a UCC contract must occur within a reasonable time after the buyer discovers or ought to have discovered the non-conformity. It is not effective until the buyer notifies the seller and the revocation must occur before there is any substantial change in the goods. Example (Contracts: UCC, Buyer Revocation) Bilbo also contracts to sell his collection of books to Frodo for a price of $1,000. Once the books are in Frodo’s possession, he examines them thoroughly and notices a defect in one of the books: it is missing 20% of the pages. He promptly informs Bilbo, thereby communicating the revocation of his acceptance. This is valid because the omission of 20% of the pages substantially impairs that book’s value. If a party unequivocally states that they are unable or unwilling to perform absent a material change in the other party’s position, this is called anticipatory repudiation. If it occurs, the non-breaching party may:

• Treat the contract as repudiated and seek damages before the time of performance is due,

• Treat the contract as discharged, • Wait until performance is due and then sue, or • Urge the other party to perform.

The breaching party can retract their repudiation if it is timely, in other words, if the party has not subsequently relied on their repudiation. If there are reasonable grounds for a party being insecure about the other’s performance, the party may request in writing adequate assurances. If the non-insecure party does not respond, the insecure party may treat this as an anticipatory repudiation. There may be instances where parties are discharged from their responsibilities and obligations under the contract. For our purposes, we will focus on accord and satisfaction, which is an agreement to discharge a liability based on terms that differ from the original amount of the contract. An accord is an executory contract between the parties to relieve a contracting party of their obligations under the contract in return for a specific act. Satisfaction refers to the satisfaction of the accord, upon which the other party is excused from further performance under the original contract. If the debtor party, i.e. the party performing the accord, fails to satisfy the accord, the

Page 60: MBE Portion UBEBB

creditor party, i.e. the party relying on performance, may sue either under the original contract or the accord terms.

Page 61: MBE Portion UBEBB

Warranties A warranty is a promise in the contract which is not a condition or otherwise. It does not necessarily concern the crux of the contract, rather it is a promise as to the suitability of the consideration exchanged. As this is more commonly used for the quality of goods, the governing law is the UCC, specifically, Article II. There are different types of warranties: implied and express. An implied warranty is one that is not necessarily contained within the contract but implied through UCC. An express warranty is one that is included within the contract.

Type of Warranty

Definition/Requirements Disclaimable? (Y/N)

Requirements for Disclaiming

Implied Warranty of Merchantability

All goods sold by merchant must be fit for their ordinary purpose

Y Must be expressly and conspicuously noted on the writing that it is “as is” or “with all faults”

Express Warranty

There is no intent needed to create an express warranty or to use words such as “I warrant” or “I guarantee” This applies when there is:

1. An affirmation of fact, promise, description, or the provision of a sample, which

2. Relates to the goods, and

3. Becomes part of the basis of a bargain

N N/A

Implied Warranty of Fitness for a Particular Purpose

This applies when: 1. The seller knows

or has reason to know of the buyer’s particular purpose for which goods are required, and

2. The buyer relies on the seller’s skill or judgment to select or furnish suitable goods

Y This can be waived through conspicuous language or waiver by the buyer

Page 62: MBE Portion UBEBB

Warranty of Title This warrants that: 1. The title shall be

good, and 2. The title is free

from security interests, liens or encumbrances

Y This can be expressly disclaimed by specific language or circumstances

Page 63: MBE Portion UBEBB

Third-Party Issues In general, a third-party cannot asset a claim for breach of contract. A third-party is defined as a party who is not in privity of contract with the parties obliged by the contract. There are different types of third parties: intended and incidental, limited by their beneficiary status. An intended third-party beneficiary is not party to the contract but has rights under the contract because parties intend their respective performances to benefit the identified third-party. An incidental third-party beneficiary is someone who happens to benefit from the contract but has no legal rights. Example (Contracts: Intended TPB) The contract for the sale of Bilbo’s house states that the final sale price is $150,000, with $7,000 of that going to pay a debt owed by Bilbo to Aragorn. Aragorn is an intended third-party beneficiary to the contract; therefore, he has an interest in the performance of both sides of the contract. Example (Contracts: Incidental TPB) The reason that Frodo wishes to buy Bag End is to set up a training school for people who want to go on hikes. Frodo sets up the school and most pupils love it. There is, however, one student that doesn’t: Pippin. Pippin is deemed as an incidental third-party beneficiary because his interest in the contract is merely incidental to the performance of both sides. In other words, he cannot sue Bilbo just because he didn’t like the way the school was structured. Only an intended third-party beneficiary can enforce their rights and sue, and this can only be done if their interests are vested. For interests, therefore rights, to be vested, the party must show that they:

• Accepted the benefit of the contract, • Upon which they detrimentally relied, and non-performance of which has

caused • Them to sue.

Once the third-party rights are vested, the contract cannot be changed or modified without the consent of the third-party. Duties to a contract are generally delegable, meaning they can be handed off from one party to the contract to a third-party. All contractual duties are delegable unless:

• The contract prohibits delegation or assignment, • Delegation is against public policy, • A personal service contract calls for very specific skills, or • The delegation materially alters the expectancy of the party wishing to delegate.

As a corollary to delegation, assignment is the transfer of rights and benefits to a third-party. Assignment generally includes a delegation of the unperformed duties by the assignor in a contract. It can be done by the assignor manifesting their intent to transfer, and the assentation of the assignment by the assignee. In doing this, unlike a regular contract, consideration is not required; however, if consideration is given, the assignment is irrevocable. A gratuitous assignment may be revoked, which makes sense because the third-party was not privy to the overall context of the contract and

Page 64: MBE Portion UBEBB

thus should not be overburdened with superfluous obligations. In general, an assignment is valid unless:

• It materially alters what is expect from the parties in the contract, i.e.: o It changes the duty of the obligor, o It increases the risk on the obligor, or o It impairs the obligor’s chance of obtaining or reducing the value of the

return performance by the obligee; • It is prohibited by law or public policy, or • It is precluded by the contract.

An assignee may sue the obligor for non-performance. As to the other rights an assignee and assignor, any defence against an assignor may be used against an assignee; and the assignee may sue the assignor for wrongful revocation or for breach of an implied warranty. Example (Contracts: Assignment) Frodo decides he wants to build a home theatre in his house, so he contracts out with Boromir to get the room built. Boromir assigns some of the work to be carried out by Tom, meaning Tom is the assignee. Though Tom was not party to the original contract, if the work is not carried out properly, Tom can still be sued by Frodo.

Page 65: MBE Portion UBEBB

Remedies The final, and arguably most pertinent aspect of Contract Law is remedies. A remedy is the court order meant to alleviate the strain to one party of another party’s non-performance or breach of contract. A remedy is either a legal one or an equitable one, or a combination of both. The most relevant type of legal remedy is called damages, which is the sum paid in order to compensate for the damage of non-performance or breach of a contractual obligation. Within damages, there are different types: • Expectation damages are damages that arise directly from the breach and are

intended to put the non-breaching party in the same position it would have been in had the breach not occurred. To recover these, the non-breaching party must show that the damage was:

o Caused by the Defendant, o Foreseeable, o Certain, and o Unavoidable, i.e. reasonable steps were taken by

the Claimant to mitigate damages. • Consequential damages are damages that arise indirectly from the breach. To

recover these, the non-breaching party must show that the potential damage: o Was reasonably foreseeable at the time of contract

formation, o Arises from the Plaintiff’s special circumstances that

the Defendant had reason to know about, and o Is certain.

You may have seen headlines of people suing large corporations for insane amounts of money; this is something specific to the US because, unlike the UK, the US court system has provision for punitive damages, which have the one purpose of punishing the Defendant. This is not wholly applicable to the study of Contract Law but will emerge with regards to Torts. As discussed above, remedies can also be equitable ones in that they achieve something fairer for both the parties. The most notable of these is specific performance which is only available if monetary damages are inadequate to compensate an injured party. Specific performance is where the court orders the breaching party to carry out a specific act, e.g. perform their end of the contract. Under UCC, a seller has different remedy options available to them. If a buyer breaches the contract, the seller may do/use the following remedies:

• Withhold delivery of the goods, • Cancel the contract, • Receive cover damages, i.e. the difference between the resale price and the

contract price if the seller resold in good faith, • Receive market damages, i.e. the difference between the market price and the

contract price, • Lost Volume Seller, i.e. the seller can recover lost profit on the sale if they

regularly engage in the sales of goods at issue at an unlimited inventory, • Stop the delivery of goods (if the buyer is insolvent and the goods are in the

possession of the carrier), or

Page 66: MBE Portion UBEBB

• Replevy (recover through seizure) the identified goods from an insolvent buyer. Replevy is allowed if the buyer was insolvent when they received the goods and the seller makes a demand for replevy within 10 days of the buyer receiving the goods.

Under UCC, a buyer has different remedy options available to them. If a seller breaches the contract, the buyer may do/use the following remedies:

• Receive cover damages, i.e. the difference between the resale price and the contract price if the buyer purchased these in good faith,

• Receive market damages, i.e. the difference between the market price and the contract price, or

• Loss in volume damages, i.e. the damages between the value as promised and the value of the non-conforming goods if the buyer kept the non-conforming goods.

Another remedy is restitution. In contrast to damages, which are loss-based recoveries, restitution is a “gains-based” recovery. In other words, restitution is awarded in order to prevent the unjust enrichment of a breach such that the breaching party confers a benefit. The amount awarded in restitution is the value of the benefit conferred. Rescission is an equitable remedy because rescission treats the original contract as cancelled. Because it is available when there is a problem with the contract formation, it will not be rescinded if:

• There is a valid equitable defence, or • The plaintiff sued for damages under the contract in a prior action.

It may help to think of “damages” as a divorce, and rescission as an annulment. And, finally, with all of damages, there is the idea of mitigation. A party cannot recover damages that could have been avoided (i.e. mitigated); and they must take reasonable steps to mitigate loss because, if they fail to do so, a court will reduce the total amount of damages by the amount that could have been avoided.

Page 67: MBE Portion UBEBB

Criminal Law and Procedure Introduction When people think about “law”, they typically think of one of two things: a slick closer or someone in court screaming “Objection, relevance?”. No? Just my family? In any event, Criminal Law is near and dear to many a law students’ hearts because it will often be their first interaction with legal education before starting law school. In this section, I want to look at the idea of the different components to a crime, before looking at the responsibility aspect of criminal analysis. Then, I’ll dive deeper into specific offences for the MBE portion of the UBE before looking at the procedural side of things, such as your Miranda warnings and Constitutional rights. Most criminal law is governed by either the common law or the Model Penal Code (MPC), so I will refer to these and identify them where necessary. Criminal procedure is governed by the Federal Rules for Criminal Procedure (FRCrP), so I will refer to this and identify it where necessary.

Page 68: MBE Portion UBEBB

Components Let’s start off nice and easy – I fortunately don’t have to use a guide for this one because I know it relatively well. For any criminal offence, there are four components: the actus reus, the mens rea, causation and concurrence (or coincidence). The actus reus (AR) is the act of the crime, e.g. the physical act of moving your hand across someone’s face. The mens rea (MR) is the mental component, in other words: what was the Defendant’s thought process before and during the commission of the AR? Causation refers to the factual or “but for” causation, combined with the legal or proximate causation. And, finally, concurrence or coincidence refers to the maxim that that the AR and MR must correspond to one another. Therefore, a useful equation to remember is this: [AR + MR] + Causation - Defences = Liability. Regarding the mens rea, the MPC outlines four broad categories of the requisite mens rea for an offence. These are:

1. Purposeful action, i.e. consciously engaging in the action, 2. Knowing commission of an action, i.e. the oblique intent to commit an action, 3. Reckless commission of an action, i.e. conscious disregard of a substantial

and justifiable risk to the extent that the action is a “gross deviation” from how a reasonable person would act, or the creation of such a risk but one where the Defendant is unaware of it solely by reason of voluntary intoxication, and

4. Criminal negligence, i.e. the failure to perceive a substantial and unjustifiable risk to the extent that the failure is a gross deviation from (read: breach of) the standard of care.

Under the common law, intention is the requisite mens rea, and this can be satisfied either directly or indirectly. There is also the idea of wilful blindness, which is where a Defendant seeks to limit liability by being ignorant. The standard in determining that a Defendant in fact had knowledge of the commission of an act is that they are either:

• Aware that certain facts are highly probable, or • They are intentionally ignorant to a certain fact.

Concerning causation, the rule is that a superseding force breaks the chain of causation. A superseding force is an event that is both independent from the event and unforeseeable. A NOTE ON ACCOMPLICES An accomplice is a party that: AR

1. Aids, abets or facilitates the commission of the crime, with MR

2. The dual intent to both assist the primary party and intend that the crime be committed.

An accomplice is also liable for all foreseeable crimes committed by the primary party. Mere presence or knowledge of a crime will not create accomplice liability, and withdrawal is a defence if done before the crime becomes unstoppable. To rely on this, the potential accomplice must:

• Repudiate (take back) the encouragement they gave the primary party, and • Neutralise the assistance they gave to the primary party.

Page 69: MBE Portion UBEBB

Specific Offences Homicide In the MBE, there are approximately 3 or 4 questions within Criminal Law that test on homicide. Homicide describes the cessation of one or more persons’ life through the action of another person or persons. You may have heard of the different degrees of murder within the criminal justice system in the US; this categorisation is something many UK students will not be familiar with. Rather unintuitively, the starting point for this section will be second-degree murder, which is defined as AR

1. The unlawful killing, 2. Of a person

MR 3. With malice aforethought, which is:

a. The intent to kill, b. The intent to inflict great bodily harm, c. The reckless disregard of an extreme risk to human life, or d. The intent to commit an inherently dangerous felony; this is known as

the felony murder rule. Example (CrimLaw: Second-Degree Murder) David and Ron are having an outdoor fight in an alleyway. David takes Ron’s head and smashes it across a wall, before stomping on his leg with a boot. Ron is taken to the hospital where he is given codeine as a painkiller. The doctor misread the chart and, unfortunately too late, discovers that Ron is allergic to codeine. Ron dies as a result of the allergy. David is liable for second-degree murder because, but for his actions, Ron would not have been in hospital to receive the incorrect medications. Furthermore, his intent was met because he, at the very least, he intended to inflict great bodily harm by smashing Ron’s head against a wall. On the other, much more serious, side of this is first-degree murder which is defined as the deliberate and premeditated killing of a person. Under the MPC, it is outlined as AR:

1. The killing of a person, MR:

2. Purposefully or knowingly, or 3. Recklessly under circumstances manifesting extreme indifference to the value

of human life. Under the felony murder rule, recklessness is presumed if the action in question is:

• Robbery, • Rape, • Arson, • Burglary, or • Kidnapping

Page 70: MBE Portion UBEBB

Example (CrimLaw: First-Degree Murder) David, irked by something Ron said seven years ago, takes a handgun and goes to Ron’s house. He knocks on the door and, before Ron can say “Come in”, he shoots Ron in the chest. Ron dies immediately. Common law manslaughter is where either the Defendant possessed the requisite mens rea during the commission of the actus reus but the law does not want to label them as a murderer, or the Defendant did not intend to kill or greatly harm the person but they died anyway as a result of the Defendant’s actions. These are also known as voluntary manslaughter or involuntary manslaughter respectively. For common law voluntary manslaughter to be valid, the Defendant must have: AR

1. Killed a person with MR

2. Adequate provocation, which is defined as: a. The provocation of the Defendant to the extent that, b. A reasonable person would have been provoked (objective), c. And there is not enough time to cool off and in fact d. The Defendant did not cool off.

For common law involuntary manslaughter to be valid, the Defendant must have: AR

1. Killed a person, either MR

2. Unintentionally, or 3. Recklessly, or 4. Under the misdemeanour murder rule, or 5. During a non-dangerous felony, or in some states 6. Through being criminally negligent.

For MPC manslaughter, the Defendant must have: AR

1. Killed a person, either MR

2. Recklessly, or 3. Been under the influence of extreme mental or emotional disturbance for which

there is a reasonable explanation or excuse. Example (CrimLaw: Voluntary Manslaughter, Common Law) David and Ron are having a friendly chat that quickly turns sour when Ron admits to sleeping with David’s partner. They take the conversation outside when David is in tears. Ron carries on, going into extreme detail about his affair with David’s partner. In a fit of rage, David punches Ron in the face, causing him to fall on the ground. His head hits the pavement and his side is punctured by a rusty piece of metal that was on the street. Ron is taken to hospital and is diagnosed with tetanus. Ron dies exactly three months later. David is liable for voluntary manslaughter because he has killed a person with adequate provocation; there was no premeditation here. Had Ron died over one year and one day after the action of being punched, David would escape liability because of the year and a day rule, which is a common law rule that a person

Page 71: MBE Portion UBEBB

cannot be convicted of a homicide for a death that occurs more than a year and a day after the acts that caused it. Other Crimes The UBE tests on other offences, but the analysis of them follows a very similar pattern, so I’ll just put them in a table:

Offence AR MR

Robbery 1. Trespassory taking and carrying away, of

2. Personal property of another, from

3. The person in their presence, by

4. The use of force or threat of immediate physical harm, with

1. The intent to permanently deprive the owner of their property

Burglary 1. Breaking and entering, of

2. A dwelling, of 3. Another, 4. At night, for

1. The purpose of committing a felony inside.

Larceny 1. Trespassory taking, and

2. Carrying away, of 3. The personal

property of another, with

1. The intent* to permanently deprive the owner of the property.

* the intent must exist at the time the property was taken

Larceny by trick 1. Obtain possession* of the personal property of another, by

* this is mere possession of the object and does not extend to the title

1. Trick or deception

False pretences 1. Obtain the title, to 2. Personal property of

another, through

1. An intentional false statement, with

2. The intent to defraud

Embezzlement 1. The conversion of, 2. Property of another,

by 3. A person who is in a

position of trust

1. Fraudulently or wrongfully

Page 72: MBE Portion UBEBB

Battery 1. The unlawful application of force, either

2. Directly or indirectly upon another person or close personal belongings, which

3. Results in injury or offensive contact

1. Intentionally or recklessly commits the AR, or

2. Absent of the intent to injure

Assault 1. An attempted battery, or

2. The intentional creation of a reasonable apprehension of imminent bodily harm to a person

1. General intent

False imprisonment (common law)

1. The unlawful, 2. Confinement of a

person, 3. Against their will,

with

1. The knowledge that the restriction is unlawful

False imprisonment (MPC) 1. The unlawful, 2. Confinement of a

person, 3. Against their will,

with

1. The knowledge that the restriction is unlawful to the extent that it substantially interferes with the person’s liberty

Kidnapping (common law) 1. The confinement, restraint or movement of a person, without

2. The authority of the law, with

1. The intent to confine, restrain or move the person

Kidnapping (MPC) 1. The abduction of another person, for

A purpose to: 1. Compel a third

person to pay ransom,

2. Facilitate the commission of a felony,

3. Inflict bodily injury or terrorise, or

4. Interfere with a government or political function

Arson 1. The burning, of 2. A dwelling, of

1. Maliciously

Page 73: MBE Portion UBEBB

3. Another.

Page 74: MBE Portion UBEBB

Inchoate Offences An inchoate (in-co-ate) offence is the term used to criminalise the preparation of commission of another offence. The most common example of an inchoate offence is the attempt, which is as it sounds. In order to be guilty of an attempt, the Defendant must show that: AR

1. They took an overt act beyond mere preparation, i.e. a substantial step, which is demonstrated by…

MR 2. The specific intent to commit a crime.

An attempt will merge with the underlying crime meaning that, if the crime is carried out, the Defendant cannot be charged with the attempt. Most states hold the view that withdrawal or abandonment of the attempt is not a valid defence if the substantial step has already been taken. However, some states allow the abandonment defence if this is done before the completion of the crime if:

• The Defendant voluntarily renounces their criminal purpose, and • They completely abandon their effort to commit the crime or otherwise prevent

its commission. Another inchoate offence is conspiracy, which is the collusion of two or more persons in order to commit a criminal offence. To be liable for conspiracy, there must be: AR

1. An express or implied agreement between two or more persons, regarding 2. The commission of an overt act in the furtherance of an unlawful objective (any

act taken by a co-conspirator is sufficient), with MR

3. An intent to enter into the agreement, and 4. The intent to pursue an unlawful act.

Regarding conspiracy, most states require the intent of all parties, whereas the minority of states require only one party’s intent. Once this is established, a conspirator is liable for the conspiracy and all foreseeable crimes committed by their co-conspirators in furtherance of the unlawful objective. As with attempt, withdrawal or abandonment is generally not a valid defence for the conspiracy itself, but it may be a defence for crimes committed by the co-conspirators after the withdrawal. The final inchoate offence to consider is solicitation, which is the commission of an unlawful act by a person under the instruction of another. For a Defendant to be liable for solicitation, there must be: AR

1. The request for another to commit or join in the commission of a crime, that 2. The other person receives, with

MR 3. The specific intent that the crime be committed.

As with attempt, this merges with the substantive crime. However, renunciation is an affirmative defence if the Defendant:

• Completely and voluntarily renounces the crime, and • Prevents the commission of the crime.

Page 75: MBE Portion UBEBB

Defences The third part of our liability equation concerns defences. A defence is a partial or full disclaimer of liability made by a Defendant in order to reduce their overall liability when a criminal offence has been committed. Duress is claimed when a person believes they were compelled to commit an act. To claim duress, a person must show that:

1. There was a threat of imminent death or serious bodily injury, to occur to 2. The defendant or another, to the extent that they were 3. Unable to avoid the harm by noncriminal conduct.

Duress is not available as a defence for an intentional killing. Mistake of fact is claimed when a person, through mistake of fact, unknowingly commits a criminal act. This is not a defence unless it negates the mens rea required for the crime. Insanity is where a person possesses the requisite mens rea whilst committing the actus reus for a crime, however, through some “mental disease or defect” (common law’s words – not mine!), the law does not want to treat them with criminal liability. To claim insanity, a Defendant must show that they have a “mental disease or defect” and satisfy any of the following four tests. Their “disease or defect” is so severe that it means:

• M’Naghten Test, i.e. they are unable to know the wrongfulness of their conduct or unable to understand the nature and quality of their act, or

• MPC Test, i.e. they are unable to appreciate the criminality of their conduct or unable to conform their action to the law, or

• Irresistible Impulse Test, i.e. they are unable to control their actions or conform their actions to the law, or

• Durham Test, i.e. that the unlawful conduct was the product of mental illness Somewhat related to insanity in its execution is intoxication, i.e. the consumption of a substance which causes an individual to act outside of the limits of their ordinary inhibitions. If a Defendant is voluntarily intoxicated, this only serves as a defence for some specific intent crimes:

• Burglary, • Assault, or • Attempt to commit a specific intent crime.

On the other hand, if a Defendant is involuntarily intoxicated, this is a defence to all crimes and is treated in the same way as insanity, i.e. through application of the same tests. The defence you’re most familiar with, however, is self-defence. Applying a common-sense approach, the definition of self-defence is a person defending themselves or another. Self-defence is a full defence, unless it is imperfect. To claim that an action was made under self-defence, a person must show that:

1. They had a reasonable belief, of 2. Imminent danger of being killed or suffering great bodily injury, to the extent

that 3. The use of deadly force was necessary.

Page 76: MBE Portion UBEBB

In a minority of states, the Defendant has a duty to retreat, but this is suspended if there was no opportunity to retreat, or the Defendant was attacked in their own home. Imperfect self-defence mitigates murder to manslaughter if the self-defence was made in good faith, but it was unreasonable.

Page 77: MBE Portion UBEBB

Criminal Procedure Criminal Procedure is the process of steps that occur from the moment of arrest up until the conclusion of a trial. There are some terms that I will gloss over because either they’re unimportant for our purposes or they will be covered in the next section, Evidence. The Fourth Amendment grants a person security from unreasonable, therefore unlawful, searches and seizures. In order to challenge a search, the individual must possess a reasonable expectation of privacy over a place or item that is searched. The Supreme Court has held that an individual can expect no right of privacy over:

• Paint taken from a car, • Bank account records, • Anything that is visible from the public airspace, • Garbage left on the kerb, • The sound over a person’s voice, • Odours, • A person’s handwriting, and • Anything that can be seen in or across areas outside of the home.

Related to search and seizure is the idea of arrest. For an arrest to be proper, the arresting officer must have probable cause, which is defined as a reasonable suspicion that a crime has occurred. This probable cause need not be first-hand knowledge and can come from an informant’s information. Unless the situation is exigent, a warrant is needed to arrest someone in or at their home. For a warrant to be valid, there must:

• Show probable cause, which in this case is defined as information that evidence will be adduced or acquired through the arrest,

• State with particularity the place and item, and • Be issued by a neutral and detached magistrate.

A seizure occurs when a reasonable person would have believed that they were not free to leave an area. When a person is seized, they may be subject to a request for information. A request for information is generally allowed any time except on “a whim” or if the officer is being capricious. Tangentially to seizure, there are the stop procedures: stop and inquire and stop and frisk (stop and search). Stop and inquire is allowed when there is a reasonable articulable suspicion that criminal activity is afoot, i.e. occurring, going to occur or already has occurred. Stop and frisk is allowed when there is reasonable suspicion that the person has a weapon. Under the plain feel doctrine, the police may only seize items that they reasonably believe is contraband or a weapon. In all these instances, reasonable suspicion is defined as the quantum of knowledge sufficient to induce an ordinarily prudent and cautious person to believe that criminal activity is at hand. Regarding evidence obtained through a warrant, the warrant itself must be valid. Otherwise, it is excluded unless it falls under one of the following exceptions:

• The search was incidental to arrest, meaning that the person may be searched upon arrest. This search is restricted to their “wingspan”,

Page 78: MBE Portion UBEBB

o If they are arrested in an automobile, the search extends to the passenger’s compartment and any containers found in the car if there is reason to believe it contains contraband;

• Plain view doctrine, which states that items may be seized if: o They are observed any of the five senses in plain view, from o A place where the officer is lawfully permitted to be, and o There is probable cause to believe that the items are evidence of a crime

or contraband; • Automobile exception, which allows a warrantless search of an automobile if

there is probable cause to believe that contraband or evidence of a crime will be found in the vehicle,

o The officer can search the entire car, and any packages, luggage, containers, etc that may reasonably contain the items for which there is probable cause

o In order to search after a traffic stop, the officer needs probable cause prior to the search in order to carry it out under this exception;

• Consent, where consent is given freely, voluntarily and intelligently. A third person with authority to do so may consent and, if two or more people share authority, any one of them may consent. However, only the non-consenting person has the authority to consent to a search of private areas, such as a bedroom or on the person;

• Stop and Frisk; • Inventory search is allowed when a person is incarcerated, or the vehicle is

impounded. This must be: o Reasonable, and o Conducted pursuant to the established police agency procedures, i.e.

they must be designed to meet the legitimate objectives of the search whilst limiting the discretion of the officer in the field; and

• Exigent circumstances, which allows a warrantless search when: o The evidence is ephemeral, o It is necessary to prevent the imminent destruction of evidence, o The police are in hot pursuit of a felon and the evidence is in plain view,

or o An emergency aid exception applies.

The Fifth Amendment protects an individual against self-incrimination when they are in a custodial interrogation.

• Custodial is when the individual reasonably believes they are not free to leave, and

• Interrogation describes a situation where an individual is likely to elicit an incriminating response.

The police must give someone their Miranda warning, which are the standard fare if you’ve ever seen any procedural show. The suspect has:

• A right to remain silent, because • Anything they can be used against them in court. • They have the right to talk to an attorney, however • If they cannot afford one, one will be provided to them.

If there was no Miranda warning, or the suspect waives their rights, any statements must be excluded through the exclusionary rule. The 5th Amendment only protects

Page 79: MBE Portion UBEBB

statements and acts that are communicative or testimonial in nature. For a suspect to therefore invoke their Miranda rights, they must make a clear and unambiguous declaration to do so. If a party invokes their right to silence, this cannot be used to incriminate them at trial, i.e. no adverse inference drawn. If a suspect makes a confession but this confession was obtained through a violation of their constitutional rights, the confession may be excluded under the exclusionary rule. Under the 14th Amendment, a violation of a suspect’s rights to induce a confession violates their right to due process if it is the product of police coercion that overbears the suspect’s will. Any waiver of rights must be knowing, intelligent and voluntary. The Sixth Amendment concerns a suspect’s right to counsel when they are formally charged. A suspect cannot be questioned without their lawyer and anything said whilst a lawyer is not present is inadmissible. This right attaches once formal adversarial judicial proceedings are commenced. Once the right is attached, any attempts to deliberately elicit statements in the absence of an attorney violate the 6th Amendment. Once a suspect invokes their Miranda or 5th Amendment rights, the questioning must stop. The police may reinitiate questioning if:

1. The suspect is re-advised of their Miranda rights, 2. They have provided a knowing and intelligent waiver of those rights, and 3. Either:

a. Counsel is present, or b. 14 days have passed since the suspect was released from custody.

If, for some reason, the counsel provided is ineffective, this is determined by the Strickland test. To establish that counsel was ineffective, the Defendant must show that:

• The counsel’s performance was deficient, and • But for the deficiency, the result would be different.

If this is established, the verdict must be reversed, and the Defendant is entitled to a new trial. The exclusionary rule states that evidence obtained in violation of the 4th, 5th and 6 Amendment rights will not be admissible at trial. All derivative evidence, i.e. evidence emerging as a result of the unlawfully obtained evidence, is also inadmissible under the fruit of the poisonous tree doctrine. The exceptions to the exclusionary rule are:

• If there was an independent, untainted source of the secondary evidence, • The discovery of the evidence was inevitable regardless of the rights violations, • The “attenuation doctrine”, i.e. the Defendant’s free will was restored through

the passage of time or intervening events; that is to say, the link between the illegal action and the tainted evidence has been reduced significantly, or

• The officers relied in good faith on a defective warrant. The limitations to the Miranda rights violations are as follows:

• A statement in violation of the Miranda rights can be used to impeach (question, discredit) the Defendant but cannot be used to impeach a third-party,

• The prosecution is not required to suppress the physical evidence because of the Defendant’s statements if those statements are voluntary, and

Page 80: MBE Portion UBEBB

• Any subsequent statement made after the Miranda warnings are given are admissible unless a previous statement was obtained using inherently coercive tactics, i.e. violating the 14th Amendment.

Page 81: MBE Portion UBEBB

Pre-Trial and Post Trial Matters Before a Defendant is formally charged, they may be subject to a line up for identification purposes if there is a victim or witness. An unnecessary line up that verges into suggestive violates a Defendant’s 14th Amendment right to due process when there is a substantial likelihood of misidentification, yet a Defendant has no right to refuse a line up. An in-court identification (by a victim, witness, etc.) is admissible if:

1. It is in court, 2. It is trustworthy, and 3. It is based on a previous transaction, i.e. it is used to back up an out-of-court

identification. At a trial, the burden of proof lies with the prosecution to prove guilt to a jury beyond all reasonable doubt. Any fact that increases the maximum penalty for a crime must be submitted to a jury and proven to that standard. A Defendant must also possess competency to stand trial, which is established if they have:

1. Sufficient present ability to consult with an attorney, and 2. A rational and factual understanding of the proceedings.

At the close of its case or the close of evidence, a Defendant may move for a judgment of acquittal. This must be granted if the evidence is insufficient to sustain a conviction for an offence, i.e. the jury would not find that the element was proven. Once a Defendant’s verdict has been reached, the principle of double jeopardy comes into play. Under the principle of double jeopardy, a Defendant is not allowed to be prosecuted twice for the same offence within the same jurisdiction. Under the Blockburger analysis, two crimes are not the same offence if each crime requires proof of a fact which the other does not. The exceptions to double jeopardy concern instances where there is either a hung jury, a manifest necessity to end the trial, or the trial was terminated at the Defendant’s request which resulted in the Defendant’s acquittal not being made on the merits of the case. In effect, double jeopardy needs the jury to have reached a verdict on a case where the facts and alleged offence are identical to the new one, and the two cases must be in the same jurisdiction.

Page 82: MBE Portion UBEBB

Evidence Introduction The end is almost near! Evidence describes the class of information that is used to prove a judgment of guilt or non-guilt. The Federal Rules of Evidence (FRE) are the primary source of statutory authority in establishing whether or not evidence is admissible. This chapter goes through the tests for relevancy, including what is and is not admissible under legal or logical relevance; then, it looks at the presentation of evidence before focusing in on hearsay and its heavily-tested exceptions. Before we get into that, let’s start by defining a witness as any person who gives a statement in a case intended to either adduce evidence or be evidence by virtue of the statement given. Witnesses can be further broken down into two broad categories: lay witness and an expert witness. An expert witness gives is someone who gives an expert testimony which is permitted when:

1. They are qualified as an expert, 2. Their testimony is helpful to the jury, 3. The expert believes their opinion to a reasonable degree of certainty, 4. Their testimony is supported by sufficient facts or data, and 5. Their testimony is based on reliable principles that were reliably applied.

Reliability is based on the Daubert/Kumho test which states that a methodology is reliable if it is:

• Founded on publication and peer review, • The error rate for that methodology is marginal, • It is testable (i.e. reproducible), and • It is a generally accepted methodology in the field.

Any witness that is not an expert is called a lay witness. Their testimony is admissible if the witness:

• Is competent to testify, and • Has personal knowledge of the matter.

They may also offer a lay opinion if: • It is rationally based on their perception, • It is helpful to give the jury an overall idea but not lead them to a legal

conclusion, and • The opinion is not based on scientific, technical or other specialised knowledge.

Page 83: MBE Portion UBEBB

Relevancy Under Rule 401, evidence is logically relevant if:

• It has any tendency to make a fact more or less probable than it would be without the evidence (probative), and

• The fact is of consequence in determining the action (material). What does this mean? Well, let’s work backwards. Example (Evidence: Logical Relevance) In a civil case pertaining to a breach of contract, Claimant alleges that Defendant failed under their obligations in the contract to use Claimant for all their computing needs. To demonstrate this, Claimant wishes to submit into evidence an email that was forwarded to them by another computer supplier contacted by Defendant. The email contains a receipt of $10,000 for five computers purchased by Defendant from Third Party. If we work on the presumption that the evidence in the form of an email is the determining factor in the action, we satisfy the second element of the test. Bearing that in mind, Defendant’s breach of contract cannot be proven without the evidence in the form of the email, meaning that the email is admissible as logically relevant evidence. It is important to note that, whilst a piece of evidence needs both probative value and material value to be admissible, the threshold for this is very low; the evidence need not possess a majority of both to be considered logically relevant. On the other side of relevance is the idea of legal relevance, which is found in Rule 403. This serves as a sort of block against logical relevance by questioning its admissibility on the bases of:

• Prejudice, • Confusion, • Misleading the jury, • Undue delay, • Wasting time, or • Needlessness.

Whilst the threshold for logical relevance is low and can go largely unchallenged, this does not immediately make the evidence automatically admissible. Legal relevance acts as a rubric for challenging evidence introduced to court on the basis of logical relevance.

Page 84: MBE Portion UBEBB

Types of Evidence A subsequent remedial measure is a measure taken after (subsequently) the act in contention in order to remedy the chance of this happening again. Under Rule 407, evidence introduced by the Plaintiff or Claimant to draw attention to this is generally not allowed in order to prove any of:

• Negligence, • Culpable conduct, • A defect in product design, or • A need for a warning or instruction.

It may, however, be admitted for other purposes. To illustrate, whilst the information may pass logical relevance and uphold legal relevance, if it nonetheless is only introduced in order to prove that a Defendant has been negligent, for example, it will not be allowed. Under Rule 408, compromise or settlement offers and negotiations are also not admissible in order to prove the validity or amount of a disputed claim or to impeach by prior inconsistent statements. Under Rule 409, offering to pay medical expenses would not be admissible to prove liability, but any related statements or factual admissions are admissible. Under Rule 410, any statements made during plea discussions, a nolo contender plea (no contest plea: a plea where the Defendant neither admits nor disputes a charge), or a guilty plea that is later withdrawn are not admissible in any subsequent civil or criminal cases. Under Rule 411, evidence of a person’s insurance status against a specified liability is not admissible to prove that the person acted wrongfully, however, the court may admit this for other purposes. Regarding the authentication of evidence, all evidence must be authenticated before being admitted.

• If the evidence is physical, this is authenticated through witness testimony or evidence that shows it has been held in a substantially unbroken chain of custody.

• If the evidence is a voice recording, it must be authenticated by anyone who has heard the person speak and have identified the recorded person as the speaker.

Under the best evidence rule, a party must provide the original document or a reliable duplicate when a witness testifies to the content of a writing or testified to knowledge gained solely from a writing. Rule 1002 also calls this the “requirement of the original document” rule. The original document is not required if any of these are the case:

• There is a reliable duplicate, or a photocopy, • All the originals are lost or destroyed, • An original cannot be obtained through the judicial process, • It was not produced after proper notice was given to the part in control and

against whom it would be offered against, or • Its use is not closely related to a controlling issue within the case.

Page 85: MBE Portion UBEBB

In general, character evidence is not admissible to prove propensity. Character evidence is evidence that speaks to a person’s character, i.e. their moral code, their subjective worldview, etc. Character evidence is admissible for non-propensity purposes. The rules for when character evidence is and is not admissible vary dependant on whether the case is a criminal one or a civil one, and further dependant on the nature of the case, and even further dependant on who is relying on the evidence. For completeness:

• Criminal Case, Defendant’s Character: a Defendant can always offer evidence of their own character; opening the door enables the Prosecution to use the evidence, but the Prosecution cannot initiate it.

• Criminal Case, Victim’s Character, Non-Sexual Offence: a Defendant can offer reputational and opinion-based evidence to show the Victim’s character, which then grounds the Defendant’s innocence. If the Defendant does this, the Prosecution can offer evidence which supports either of these.

• Criminal Case, Victim’s Character, Sexual Offence: a Victim’s sexual history or predisposition is never admissible. All that is admissible is character evidence to prove that the Defendant was not involved in the crime, or sexual relations between the Victim and Defendant, or if the exclusion of the evidence violates the Defendant’s constitutional rights.

• Criminal Case, Victim’s Character, Homicide: a Victim of a homicide’s character is only admissible if the Defendant claims that the Victim was aggressor. If this occurs, the Prosecution can only introduce character evidence to show the Victim’s character as contradictory to the Defendant’s allegation.

• Civil Case, Non-Sexual Misconduct: character evidence to prove a propensity of behaviour is generally not admissible in a civil case

• Civil Case, Victim’s Character, Sexual Misconduct: this is admissible if the probative value of the evidence substantially outweighs the danger of harm to any victim and unfair prejudice to any party. The victim’s sexual history or predisposition is only admissible if the victim has placed it in controversy.

A habit may also be admitted as evidence to show that a person acted in accordance with a routine practice. A habit is defined as a regular response to a repeated situation. During a trial, if character evidence is being examined, there are limits to the procedure based on the phase of argument. During direct-examination, i.e. examining your own witness, only reputational and opinion-based evidence is up for questioning. During cross-examination, i.e. examining the other side’s witness, reputational, opinion-based and the examination of specific acts is allowed. With that said, however, specific bad acts alone cannot be admissible to show propensity; they can only be admissible to show:

• Motive, • Identity, • Absence of Mistake/Accident, • Intent, • Common Plan/Scheme, • Opportunity, or • Preparation.

Page 86: MBE Portion UBEBB

Except in cases of sexual assault or child molestation, in which case the Defendant’s character pertaining to specific bad acts is admissible if the Defendant is accused of that conduct. In order to offer this evidence, the party relying on its introduction must show:

• By a preponderance of evidence that the prior act was committed, and • That the probative value of the evidence substantially outweighs the danger of

unfair prejudice. If a Defendant is being impeached, prior convictions involving dishonesty are always admissible to impeach unless the record was expunged. Any other misdemeanours are always inadmissible. For felonies involving dishonesty, the rules are varied:

• Civil or Criminal Cases: the evidence is admissible if the witness is not the Defendant, but this is subject to the Rule 403 exclusions.

• Criminal Cases: the evidence is admissible if the Defendant is a witness but only if the probative value of it outweighs the prejudicial effect it may have on the trial.

• 10-Year Exception: if 10 years have passed, either from the conviction or release from jail, whichever is later, the conviction is inadmissible as character evidence unless the probative value of its inclusion outweighs the prejudicial effect it may have on the trial.

A witness’s credibility may be attacked on cross-examination with specific instances of conduct, i.e. prior bad acts, only if the conduct is probative of the witness’s character for untruthfulness. Extrinsic evidence, i.e. evidence adduced from external sources and not those relied upon at trial, is never admissible to attack or support a witness’s credibility. Regarding prior inconsistent statements, i.e. a statement made by a witness that contradicts or is inconsistent with one previously made, these are admissible to impeach a witness. Extrinsic evidence of that statement, however, is only allowed if the evidence is not offered against a party opponent and:

• It is relevant to the material issue at trial, and • Proper foundation is shown, i.e. the witness is first given an opportunity to

explain or deny the statement, and an adverse party is given an opportunity to examine the witness about it.

A witness’s credibility may be attacked or supported by either:

• A testimony about reputation for having a character of untruthfulness/untruthfulness, or

• A testimony in the form of an opinion. In effect, if a witness’s credibility is being debated at court, their credibility may only be rebuked by a testimony from a third-party regarding their untruthfulness or attested to by a third-party giving their opinion on the witness. If the third-party is giving a statement to testify to the witness’s truthfulness, this can only be made after their credibility has been attacked. This is further limited by the fact it does not allow for extrinsic evidence to form a part of the testimony in support of or against the witness. A witness’s ability to observe, remember or relate facts accurately (or inaccurately, for that matter) may be attacked during impeachment.

Page 87: MBE Portion UBEBB

Presentation of Evidence Courts have further rules in how evidence must be presented, i.e. in what manner the evidence should be brought to the attention of the court, including the witnesses, the parties, the judge and the jury. The first of these ways is through what is known as a “refreshing recollection”. A refreshing recollection is a document used to help refresh a witness’s recollection of or to when the witness once had personal knowledge of the matter but is unable to recall it. Only the opposing party may offer into evidence the item that is used. Another way is through what is known as a judicial notice, which allows a fact to be introduced into evidence if the truth of that fact is so no notorious or well-known that it cannot be reasonably doubted. In other words, if there is a fact so indisputable, the court will make a judicial notice of it to allow its presentation as evidence. For a court to make judicial notice over a piece of evidence, it must be either:

• A commonly known fact in the community, or • Readily capable of verification to the extent that it cannot be reasonably

questioned. For criminal cases, the court must instruct the jury that it may or may not accept the noticed fact as conclusive. For civil cases, the fact is deemed dispositive thus the court must instruct the jury to accept the notice as conclusive.

Page 88: MBE Portion UBEBB

Hearsay Alright, this is by far the most heavily tested area of Evidence in the UBE. Studicata puts hearsay, and its exceptions, as getting 6 or 7 of the 25 Evidence questions. Not to mention that, if there is a CrimPro or CivPro essay, they’ll most likely include hearsay. Needless to say, it’s important. Hearsay is an out of court statement that is offered to prove the truth of the matter asserted. The general rule is that hearsay is not admissible, however, the UBE will always test on the exceptions to the rule. A non-verbal act can be considered as a statement if it is intended as an assertion. Finally, if there are multiple assertions within one statement, each of those statements must fall within an exception to be admissible. Things that are not considered hearsay, i.e. non-hearsay, are admissible if they are offered to prove something other than the truth of the statement. Common non-hearsay statements include:

• Attestations that a statement was made (rather than attestation as to the veracity of the statement),

• Circumstantial evidence as to the declarant’s state of mind, and • Other verbal acts of independent legal significance.

Under the FRE, non-hearsay statements are:

• Admissions by a party opponent, i.e. any statement offered against a party which is made by that party,

• Prior statements of identification, i.e. the declarant identifies a person as someone they perceived earlier, and

• Prior statements made under oath, which are admissible if: o The declarant testifies, o The declarant was subject to cross-examination, and o The prior inconsistent statement was given under oath.

There are, let’s say, a lot of hearsay exceptions and there is no real formula as to which ones will show up on the UBE and in what capacity. I’m going to make a table which you can upload, copy, alter or do whatever you want to. This table will use the following fact pattern: Amber and Brody are friends who work in an office together in State D, where Brody lives. Amber lives in State E. Their co-worker, Cam, sees and hears a skirmish before rushing over to them and seeing Amber slap Brody. Brody gets a deep scratch on their cheek and has to get stiches, which means that they are permanently scarred. Brody’s side-hustle is at a modelling job where they lost a contract worth $78,000 due to the scratch. Brody sues Amber in federal court. Venue and jurisdiction are proper; the case proceeds to trial where Charlie is a witness for Brody.

The Hearsay Exceptions Name Definition Example Allowed because

Present Sense Impression

A statement describing an event made by the declarant while observing the event or

“…After running to them, I was sure that

This is allowed because Charlie is giving their

Page 89: MBE Portion UBEBB

immediately after; within a few minutes is fine as well

Amber was going to hurt Brody more”.

impression as at the time of the act.

Excited Utterance A statement made concerning an exciting or startling event, made while the declarant is still under the stress of the event; a slight delay to this is fine

“…I yelled at Amber and said, “No, stop, you’re going to stab their eye out!””

This is allowed because Charlie was startled by the events unfolding and made a statement under the stress of the act.

Business Records

These are admissible if they are:

1. Records of the events, conditions, opinions, diagnoses, that are

2. Kept in the regular course of business, which were

3. Made at or near the time of the matter described, and

4. Made by a person with knowledge, and doing so was

5. A regular practice of the business, as long as

6. The opponent party does not show a lack of trustworthiness.

This is not the first time that Amber has had an incident with a fellow member of staff; business incident reports show that this in fact the third time.

---

Statement of Medical Diagnoses/Treatment

These are admissible if: 1. The statement

was made for medical diagnosis or treatment, and

2. The statement describes the medical history or symptoms that the subject has or has had.

Brody is suffers from haemophilia, meaning they bleed excessively.

---

Page 90: MBE Portion UBEBB

Statements of Mental, Emotional or Physical Condition

These are admissible if the statement is about a subject and of a then-existing state of mind, or it speaks to their emotional or physical condition. Statements made of memory or belief are not admissible unless it relates to the validity or terms of the declarant’s will, i.e. what they intended to do.

--- ---

Dying Declaration (only to be used in civil or criminal homicide cases)

To be admissible, the: 1. Declarant must

be unavailable, 2. Statement was

made under a sense of impending death, and

3. Statement concerned the circumstances or causes that put the declarant in the position of impending death.

--- ---

Statement Against Interest

A statement against a person’s own penal, proprietary or pecuniary interest is only admissible if the declarant is unavailable. If this is in a criminal case, the statement must be supported by corroborating circumstances that clearly indicate its trustworthiness.

--- ---

Government or Public Records

The following records are admissible in court:

• A record describing policies and

--- ---

Page 91: MBE Portion UBEBB

practices of a public office,

• Observations made by someone in accordance with their duties by law, unless it is a criminal case and the observation is a police report, or

• Factual findings made during an investigation pursuant to authority granted by law, if the case is a civil one or where the government is a criminal Defendant.

The records will not be admitted if the opponent party shows trustworthiness.

Past Recollection Recorded

This is a record made on a matter the witness once knew about but now cannot recall well enough to testify fully and accurately. These are admissible if:

1. The witness had personal knowledge at one time, and

2. The writing was made or adopted by the witness, which was

3. Made while the event was still fresh in the witness’s mind, and it was

4. Accurate, however

--- ---

Page 92: MBE Portion UBEBB

5. The witness can no longer remember the event.

The record can read into evidence but can only be presented as an exhibit offered by an adverse party to the party relying on it.

The final procedural note on hearsay concerns the 6th Amendment: it gives a criminal Defendant the right to confront any witness against them. The use of an out-of-court statement violates the 6th Amendment when:

• The statement is testimonial, • The declarant is unavailable to be cross-examined at trial by the Defendant,

and • The Defendant did not have the opportunity to cross-examine the declarant

before the trial. For a statement to be testimonial for purposes of the 6th Amendment, it is either:

• A statement made to a grand jury, or • A statement made to police whose primary purpose was to collect testimony to

be used at a later trial o A statement made to assist the police in an ongoing emergency,

therefore, is not considered testimonial. For a circumstance to be an ongoing emergency for the purposes above, it is determined by:

• The nature of the dispute, • The potential harm to the victim, • The threat to additional, identifiable victims, • The generalised threat to the public, • If there was a weapon at concern, and • Whether the suspect is at large, located or otherwise not apprehended.

Page 93: MBE Portion UBEBB

Privileges Certain privileges are afforded to persons connected to witnesses. If you’ve ever seen any American drama show on TV, you might have seen the plotline of two people getting married because spouses “can’t be forced to testify against one another”. Turns out, that’s a gross over-simplification of a nuance – who would’ve thought it? But it does provide a nice segue for me to talk about the first of the privileges: spousal immunity, which is the idea that spouses cannot be used to incriminate or implicate one another. Spouses actually possess two types of privilege: communication privilege and testimonial privilege. With communication privilege, spouses can protect everything said between one another from the moment of marriage. It extends even after divorce or death, but not annulment since that action erases the marriage as though it never happened. Obviously, if the spouses are suing one another then that privilege cannot be invoked. This privilege applies in both criminal and civil cases, but it does not apply if for example a spouse commits a crime against the other spouse or any of their children. The more commonly known type of spousal privilege is testimonial privilege, which is held by the “witness-spouse”, i.e. the spouse who is not party to the case. This allows a witness-spouse in a valid marriage to refuse to testify against the party-spouse in a criminal case. The other type of privilege is something we touched on in CivPro and is called attorney-client privilege, which protects confidential communications between an attorney and their client if it was intended to be confidential and it was made to facilitate legal services. In this instance, the client holds the privilege and may waive it through intentional disclosure to a third-party. The “intentional” part is the key here because the privilege is not considered waived if the disclosure to the third-party was an inadvertent one. If the client is a corporation, the privilege is usually only applicable to high-level employees, meaning executives or decision-makers. The privilege cannot apply when:

• It is sought to further a crime or fraud, • The litigation is related to a breach of duty between an attorney and their client,

or • Joint clients are later involved in civil litigation against one another.

Related to attorney-client privilege is the attorney work product doctrine which protects from disclosure all materials prepared by an attorney or their agents in anticipation of or during litigation. The one exception to this is that the privilege is waived if the opposing party shows both a substantial need for the materials to exist and that a substantial equivalent cannot be obtained without undue hardship. The final privilege of concern is the physician-patient privilege, which protects patient communication if made to a physician for the purpose of medical diagnosis or treatment. However, the federal courts do not recognise this privilege. That said, state law governs privilege for civil cases in federal court if state law has supplied the rule of decision, i.e. “the law applied by federal courts”.

Page 94: MBE Portion UBEBB

Real Property Introduction Can you taste that? That sweet smell of freedom telling you that the end is nigh? This chapter will cover land law (the law of real property) as necessitated by the UBE. The concepts are fairly similar to the UK’s, but there are some key things that the UBE tests which generally haven’t been covered in great depth from the UK’s LLB perspective. To start, we’ll discuss the ownership of real property, before moving on to the rights a legal person has over real property; then, we’ll look at the different types of real estate contracts, before going to discuss mortgages and then concluding with titles. If you’re dismayed by the prospect of land, try and think of the whole module as a puzzle that’s just begging to be solved; each bit of information is a clue, and the more clues you have, the clearer it gets.

Page 95: MBE Portion UBEBB

Ownership of Real Property When we discuss ownership of real property, we must first establish what is meant by the term. Real property obviously means the land in question, but it also includes anything attached to the land, such as buildings, annexes, poles, walls, fences, etc. The term is broad enough to further make provision for the airspace above the land and the ground below the land, but these are only to the extent necessary for ordinary use and enjoyment of the land. So, when someone “owns land”, they own more than just the space within a jurisdiction. In addition to that, when we own land, we actually own an estate in land. An estate is ownership of a property for a period, therefore a PRESENT ESTATE is an estate that entitles the owner to possession of land in the present. Within the different estates in land, there are: fee simple estates, life estates, and leaseholds.

• A fee simple estate is an estate whose period of ownership carries on indefinitely;

• A life estate is an estate whose period of ownership carries on until the death of the owner, upon which the property transfers to another party which is specified by deed. A life tenant is responsible for the ordinary expenses and taxes for the property during their lifetime and thus cannot commit waste, i.e. cause the property to decrease in value;

• A leasehold estate is an estate whose period of ownership carries on for a specified term.

If an estate is a fee simple one, it can be a fee simple absolute, which enables the owner to have absolute, unconditional ownership of the property. Else, it could be a fee simple defeasible, which is an estate in land with express conditional language that enables the grantor (the person granting ownership in the land) to terminate the grantee’s ownership upon the occurrence or non-occurrence of an event or a condition. A fee simple determinable is a type of fee simple defeasible; if an interest in land is a fee simple determinable, there is the possibility of reversion, i.e. its ownership will automatically revert to the grantor unless a specific condition is fulfilled. Another type of fee simple defeasible is a fee simple subject to a condition subsequent, which basically means that the ownership of the land may revert to the grantor, but the grantor must affirmatively asset their right of re-entry as failing to do so enable the ownership of the land to continue as before. It may help for you to look at the Venn diagram below in order to illustrate the levels of ownership:

Ownership in Land

Fee Simple Estate

Fee Simple Defeasible

Fee Simple Determinable and

Fee Simple Subject to Condition

Subsequent

Page 96: MBE Portion UBEBB

The final type of fee simple defeasible is called a fee simple subject to executory interest, which in effect is the same as a determinable or condition subsequent, only instead of the ownership reverting to the grantor, a future interest is reserved by the grantor on behalf of a third-party. Let’s do some examples. Example (Real Property: Fee Simple Absolute) Eric buys a house from Kenny and is entitled to do whatever he wants with that house. This is a fee simple absolute. Example (Real Property: Fee Simple Determinable) Eric buys a house from Randy, but Randy says that Eric must use at least 0.5 acres of the land to grow marijuana. This is a fee simple determinable because Eric’s enjoyment of the land depends on his adherence to growing the marijuana. Should Eric die, the person or persons to whom the land is bequeathed will be burdened by the condition. Example (Real Property: Fee Simple Subject to a Condition Subsequent) Eric buys a house from Randy, who says that he should use at least 0.5 acres of the land to grow marijuana. This is a fee simple subject to a condition subsequent because, while Randy told him that Eric should grow some marijuana, if Randy does not assert this right, there is nothing to prevent Eric from otherwise enjoying the land as he pleases. A FUTURE INTEREST is an interest in land that does not include the right to present possession or enjoyment of the property, in other words, a person may own property but does not enjoy the rights in land that a present owner does. There are six types of future interests, and these are also dependent on whether the person in question is a grantor or grantee. Before I proceed, it is important to note that a remainder is an interest in land held by a person after the original grantee’s use has expired. Consider the table below:

INTEREST DEFINITION PAIRS WITH ENJOYED BY

Possibility of Reverter

A future interest that states ownership reverts back to the Grantor if a condition is not satisfied

Fee simple determinable

GRANTOR

Right of Re-Entry A future interest that reserves the right to re-enter and take ownership of the property which can be asserted should a

Fee simple subject to a condition subsequent

Page 97: MBE Portion UBEBB

condition remain unsatisfied

Reversion A future interest that reverts the property back to the grantor if the grantor grants ownership of the estate to the grantee for a lesser estate

Life estate, leasehold

Vested Remainder A future interest held by an ascertained person that is not subject to a condition precedent

Life estate, leasehold

GRANTEE Contingent Remainder

A future interest held by either an unascertained person or an ascertained person with a condition precedent

Life estate, leasehold

Hopefully that isn’t confusing you too much; I’ll get onto some examples towards the end of the section but there’s a lot more to get through because Property is one of those modules where you can’t really see things in action until you get a more thorough picture. Let’s discuss the lovely world of CO-TENANCY, which is shorthand for two or more people who enjoy the use of property. There are two types of co-tenancy: tenancy-in-common (TiC), and joint tenancy (JT). Tenancy-in-common is the default estate that is created when land is conveyed to two or more people unless there is express language which states the parties wish to create a joint tenancy, thus enabling survivorship, or the land is conveyed to a “husband and wife” party, in which case it is a tenancy by entirety. Under a TiC, each party holds an undivided interest in the property and has the right to use and enjoy the entire property. This interest passes through bequeath or intestacy. The other type of co-tenancy is a joint tenancy, which is created when the four unities of property are present:

• Time, i.e. the interest must be obtained by the tenants at the same time, • Title, i.e. the interest must arise out of the same instrument, • Interest, i.e. the tenants must have the same interest in the property, and • Possession, i.e. the tenants must have the right to possess the whole property.

As mentioned above, to form a JT, there must be a clear express intent to create one with the right of survivorship. The right of survivorship states that, if one tenant dies, their interest in the property is survived by the other tenants. To end a JT, it must be severed. Through severance, a joint tenant unilaterally transfers their interest in the property. Since the interest is no longer united, the property falls back as a TiC. Severance, however, doesn’t automatically make a JT a TiC; the tenancy can still proceed as a JT if there are more than two tenants if the remaining tenants wish to proceed as joint tenants. If a joint tenant wants to take out a mortgage on their interest,

Page 98: MBE Portion UBEBB

it does not sever joint tenancy in lien theory jurisdictions, but it does sever the joint tenancy in title theory jurisdictions. A lien theory jurisdiction is a jurisdiction where the title to the property is held in the name of the borrower. The lender has a security interest (a lien) to the property; therefore, if the property is sold, the lien is paid off and the title is released to the borrower. A title theory jurisdiction is a jurisdiction where the title to the property is held in the lender’s name until final payment is made. Once final payment is made, the title is passed on (re-conveyed) to the borrower. An out-of-possession co-tenant is a co-tenant who no longer retains possession of the property. They typically do not have a right to receive rent unless they were wrongfully ousted. An in-possession co-tenant is a co-tenant who presently retains possession of the property. Generally, they are not liable for rent for out-of-possession co-tenant’s use of the entire property. However, if the property is rented out to a third-party, the out-of-possession co-tenant is entitled to the fair share of rent paid by that third-party. A co-tenant is entitled to reimbursement for the costs of necessary repairs unless they were wrongfully ousted. However, unless agreed beforehand, they are not entitled to reimbursement for improvements to the property. That said, the co-tenant who improves the property is entitled to the downside risk or upside gain when the property is sold. LANDLORD-TENANT LAW concerns the legal relationship between two parties engaged in a hierarchical connection over a leasehold interest. A leasehold interest grants the tenant the present possessory interest in use of the land for a limited period, which can take the form of a tenancy for years, i.e. it lasts for a fixed period and is automatically terminated after the period. It can also take the form of a periodic tenancy, i.e. it continues for a specified period of weeks, months or years until it is terminated by proper notice. It is created either:

• Expressly, • By implication, or • By law.

To terminate a periodic tenancy, there is a written notice requirement which must be made at least a full period in advance. If the tenancy is a yearly one, the notice period is 6 months. The final type of leasehold interest is a tenancy at will which is created by express agreement and does not generally specify the length of the tenant’s duration. This type of tenancy continues until either party terminates it; termination is usually subject to a notice combined with reasonable time to exit the premises; however, some states do not have a notice period. Over the leasehold, the duty to pay rent runs with the land. If the tenant enjoys the land but does not pay the specified rent, the landlord can either evict (eject, remove) the tenant or allow the tenant to stay on the land and subsequently sue for damages. If the tenant abandons the property (leaves the property without proper notice and refuses to pay rent), the landlord should take steps to mitigate their losses depending on the law of the state. Note, however, that this is not a common law duty but most states do have a duty to take reasonable mitigation steps. On the flipside of abandoning is a holdover tenant. A tenant is a holdover tenant if they refuse to leave the property after the expiration of the lease. If a tenant holds over, the landlord may either evict them or hold the tenant over. When the latter of these is done, it is implied

Page 99: MBE Portion UBEBB

that a month-to-month tenancy is created with terms identical to the original. If, before the end of the lease, the tenant was informed that the rent will increase, that higher rent will still apply. As to how long this can stay in existence: the common law states that the new tenancy is equal to the period of the original one, however, the contemporaneous practice is that a month-to-month tenancy is created. The landlord also has a duty of habitability, which means that the place is reasonably suitable for human needs. This is implied in every residential lease and some courts may also impose a warranty of habitability for commercial leases in limited circumstances. If the warranty is breached, the tenant may either:

• Move out and terminate the lease, • Withhold or reduce the rent, • Repair the issue and deduct the cost, or • Remain on the premises and sue for damages.

In a residential lease, the landlord has an implied duty to repair common areas and to warn the latent defects that create a risk of serious harm that the landlord knows or ought to know of. Rather unfortunately, for commercial leases, the landlord has no duty to repair. Every lease also enjoys an implied warranty of quiet enjoyment which has the effect of preventing the landlord from interfering with the tenant’s quiet enjoyment of and possession of the property (for the contracted period, obviously). Therefore, when this is breached, the landlord may have effectively committed constructive eviction, meaning that:

• The landlord breached a duty to the tenant, which • Caused a loss of the substantial use and enjoyment of the premises, to the

extent that • The tenant gave the landlord notice of the condition, upon which • The landlord failed to remedy that condition within a reasonable time, causing • The tenant to vacate the premises.

If constructive eviction occurs, the tenant or tenants may terminate the lease and seek damages. An assignment of a lease is where enjoyment and use of the property for the specified period is assigned by the tenant to another person who is not a tenant, in effect, the tenant transfers all their remaining leasehold interest over the property to another person. Leases may be freely assigned unless the lease states otherwise. An assignment can never be for a longer period than the remaining lease term, and a landlord waives the right to enforce a provision prohibiting the assignment if they accept rent from the assignee. Under the doctrine of privity of estate, i.e. the legal relationship between two parties when their relationship concerns one estate in law, the assignee is liable to the landlord for rent and all other covenants that run with the land. Under the doctrine of privity of contract (already covered in Contracts and Sales), the original tenant remains liable for rent should the assignee fail to pay. An assignment is different to a sublease, which is where the tenant transfers only some of their remaining lease interest; in effect, some enjoyment of the property for some duration. Unless the lease says otherwise, a tenant can freely sublease. Because there is no privity of estate, the subleasee is not liable to the landlord for rent or other covenants that run with the land; this means that, under the doctrine of privity of contract, the original tenant is still liable for rent to the landlord.

Page 100: MBE Portion UBEBB

To end a lease early, the tenant must surrender. Unless the agreement says otherwise, the landlord must clearly accept this; therefore, any attempt to end the lease early does not constitute surrender unless the landlord clearly accepts this. If the landlord does not accept it, the tenant has “abandoned” the property and therefore is liable for damages, but again the landlord must take steps to mitigate the amount of damages as they will not be awarded for reasonable steps that the landlord failed to take.

Page 101: MBE Portion UBEBB

Rights in Real Property A covenant is a rule that applies to use of the land which obligates the holder of the title to do something or refrain from doing something. There are two aspects to covenants, the burden and the benefit, and how these apply depend on what covenant is in contention. For a restrictive covenant, i.e. something restricting use of the land, the benefit is attached to the land (it runs with the land) and the owner has a right to enforce the benefit if:

• It is in writing, which satisfies the Statute of Frauds, • Its intent is made clear that it will run with the land, • There is vertical privity, i.e. the successor holds the same and entire interest

that was held by the predecessor, and • The covenant touches and concerns the land, i.e. makes the land more useful

and valuable. For a positive covenant, i.e. something that obliges a positive action over the land, the burden is attached to the owner of the land at that time. In other words, it does not run with the land. The owner of the land at that time can enforce the burden by:

• The methods described for a restrictive covenant, and by • Horizontal privity between the original parties, as long as • The new owner has been given notice of the covenant, which can be made by:

o Actual notice, o Notice through record, or o Notice on inspection or inquiry of the land.

If a party breaches a covenant, the remedy is damages. An equitable version of a covenant is called an equitable servitude, which is a covenant that is enforced through equity if the burdened estate had notice of the covenant irrespective of if it is a positive or restrictive one. To enforce the benefit, the owner must show that:

• There was writing which satisfied the Statute of Frauds, that possessed • An intent for servitude to be enforceable, and • The covenant touches and concerns the land.

To enforce the burden, the owner must show: • The methods for enforcing the benefit, and • Notice given to the new owner, which can be made by:

o Actual notice, o Notice through record, or o Notice on inspection or inquiry of the land.

An easement is a non-possessory interest in the use of someone else’s land. For example, a property (A) that is enclosed by another property (B) will require an easement for non-possessory use of B to access A. In the creation of an easement, there is typically a dominant estate; which is identified as belonging to the party gaining the benefit of the easement; and a servient estate, which is identified as belonging to the party burdened by the easement. An easement by grant is an easement which is made by the grantor to the grantee and requires:

• Writing signed by the grantee, who is the owner of the burdened property, that • Reasonably identifies the parties and the land, and

Page 102: MBE Portion UBEBB

• Indicates the grantor’s • Intent to convey the easement.

An easement by implication is an easement that is created through non-contractual means. This requires:

• A single tract of land divided by a common owner, where a • A pre-existing use by the owner was established prior to division, but there is • A continuous and obvious indication that the use was meant to be permanent,

and the use • Affects the value of the land conveyed, to the extent that • The use is reasonably necessary for the owner’s use and enjoyment of the land

conveyed. An easement by necessity is an easement created through necessary use of the servient land. It requires:

• Land owned by one owner that is subdivided, to the extent that • Access to the easement is essential to the use of the property because there

exists no ingress (right entry) nor egress (right of exit). An easement by prescription is an easement created where the landowner (of the servient estate) fails to bring an action against those using the land for the period of the statute of limitations for land actions. This is created if:

• The possessor’s use is open, continuous and hostile, and within • The statutory period.

An easement may be terminated by any of the following methods:

• Estoppel, when the servient owner reasonably relies on the assurance that the easement will not be enforced,

• Termination of the necessity, e.g. construction of a road for an enclosed piece of land,

• Involuntary destruction of the servient estate, • Condemnation of the servient estate, e.g. a government taking, • Written release, • Abandonment, i.e. physical action to show intent to never use the easement

again, • Merger of the properties, though if they are separated again the easement does

not automatically revive, or • Prescription, i.e. the antithesis to a prescriptive easement.

A licence over land is not an easement. It is a privilege to use another’s land for a particular purpose. It must be in writing and can be revoked at any time, however, the licensor may be estopped from revocation if the licensee invested substantial money or labour in reasonable reliance of the licence. The final right to discuss is the right of profit, which gives the proprietor the right to take resources such as petroleum, minerals, timber, etc from the land of another and profit off them. The profit proprietor may be different to the landowner; the facts will say so.

Page 103: MBE Portion UBEBB

An item affixed to the land that then becomes part of the parcel of land is called a fixture. Whether or not an item is a fixture is a matter of objective intent, which is determined by considering:

• The nature of the item, • The way it is attached, • Any damage that would result if it was removed, and • The extent to which the item is adapted for use on the land.

If a tenant is a trader or business, an item attached for a tenant’s trade or business is not considered a fixture unless its removal causes substantial damage to the property. In this instance, it may also become a fixture if it is not removed before the end of the term of the lease. Zoning describes the method of planning in which a bureaucracy divides lands into areas (zones), where certain land uses are permitted or prohibited. Zoning laws, or land-use regulations, are allowed when it reasonably protects the health, safety, morals or general welfare of the community. An exception to a zoning ordinance is called a variance, and there are two types: a use variance, which alters the purposes for which land can be used, and an area variance, which alters the boundary subject to the ordinance. A variance application may be granted if the property owner shows that they will suffer hardship as a result of the ordinance, and the variance will not damage or harm the public owner. There may also be instances where a new zoning law or ordinance is enacted that makes non-conforming a landowner’s use of land that was otherwise non-violating. Under the doctrine of previous non-conforming use, a landowner may continue to use their land in violation of a later enacted zoning law, so long as the prior use was legal. Insubstantial changes and reasonable alterations to repair the property are also permitted, but substantial changes to the property are not allowed. Finally, the non-conforming use must be continuous otherwise it wil no longer be protected.

Page 104: MBE Portion UBEBB

Real Estate Contracts For a valid contract for the conveyance of land, the contract must follow certain criteria:

• It must be in writing, to satisfy the Statute of Frauds, • It must describe the property, • It must identify the parties, • It must contain the purchase price, and • It must be signed by the grantor or the grantee, i.e. whoever the contract is

being enforced against, e.g. if it is for the purchase, it is signed by the grantee; if it is for the sale, it is signed by the grantee.

In addition, real estate contracts require the title to be marketable, meaning it requires the seller to have proof of title to all the property being sold and that there are no undisclosed third parties with undisclosed interests in the title. In short, the title must be free from any cloud or subject to any adverse claim. The title is unmarketable if the property has a substantial defect, meaning a defect in the record chain, or encumbrances such as some mortgages, liens, restrictive covenants, etc. If the seller cannot convey a marketable title at closing, the buyer may withdraw from the purchase without penalty. A buyer may waive the requirement for marketable title because the benefit of the nonmarketable title is for the buyer. If the buyer does waive this, the seller cannot cancel the contract. If the seller breaches the warranty of marketability, the damages equates to a reduced value of the land thus a lower purchase price. Under the merge doctrine, once the deed is accepted and delivered, the sale contract merges with the deed which has the effect of extinguishing any rights to sue under the contract, meaning the buyer can only sue pursuant to the deed. Once a contract is signed, the land itself exists in a limbo of sorts between owners. Under the doctrine of equitable conversion, the ownership is split between the buyer and seller. The buyer possesses equitable ownership of the property, whereas the seller has legal ownership, meaning they hold the property in trust for the buyer. In most states, the risk of loss transfers to the buyer upon signing the contract, meaning it is the buyer’s responsibility to mitigate the loss that may subject the property before closing. In the minority of states, the seller bears the risk of loss unless the buyer has possession or title to the property at the time that any loss occurs before closing. Whilst the contract may specify a closing date, the time for performance can be changed if one of the parties requests it. The one exception to this is if the contract contains a clause known as a “time of the essence” clause which effectively means that the buyer is in material breach if they fail to perform their side of the contract by or on the closing date. The seller may keep the down payment or deposit as liquidated damages for the breach. If the house is a new build, there is a home builder’s implied warranty in the contract. This protects the purchasers of newly built homes against latent defects and warrants that the home is safe and fit for human habitation at the time of sale. A latent defect is a defect that could not have been discovered by reasonable inspection. There may also be an option in the contract in the form of a right of first refusal, i.e. a potential future right over the property that is contingent on the property being put

Page 105: MBE Portion UBEBB

on the market. If there is a right of first refusal, the property must be offered first to the identified party before its sale to any other buyer.

Page 106: MBE Portion UBEBB

Mortgages A mortgage is a security over a property. A borrower may borrow money from a lender to fund the purchase of a house; in return for security, the borrower enjoys the use and possession of the interest whilst the lender possesses the title up until the mortgage is paid off. The effect of creating a security over property is called creation of a charge, and the charge burdens the borrower. For a mortgage to be valid, it must:

• Be in writing, • Be signed by the chargee, and • Reasonably identify the parties and the land.

For most real estate transactions, we often see them as: Buyer wants house à Buyer gets a mortgage to cover the cost of purchase à Buyer uses money to purchase house and pays back the bank a monthly fee to pay off the mortgage à After paying the mortgage, the Buyer owns the house fully. In fact, it is a little more complex, but marginally so. A purchase money mortgage works like this: Buyer wants house à Buyer applies for mortgage à Bank/Lender buys house, creating a security à Lender creates a charge to enable Buyer to enjoy use and possession of the interest of the house à Buyer pays, as failure to do so will result in repossession. A junior mortgage is a mortgage that is subordinate to a prior mortgage, often called a senior mortgage. A future advance mortgage is a loan by which the lender may provide future payments under the original loan, meaning a security is granted over another piece of property. A deed of trust is similar to a mortgage but involves three parties: the borrower, the lender, and a third-party who holds the title of property as trustee until the loan is paid off. The assumption of a mortgage describes the conveyance of the terms and balance of an existing mortgage to the purchaser of a financed property (A transfers their mortgage to B, who has “assumed the mortgage”). This can be express or implied. An express assumption of mortgage simply requires an express agreement, whereas an implied once can be found in the absence of an express agreement, combined with the conditions of the grantee paying the seller only for the equity along with the grantee continuing to make payments to the lender on the balance of the mortgage. Foreclosure extinguishes junior mortgages and is when the proceeds from the sale are used to pay off debts in the following order:

1. Attorney fees and sale expenses, then 2. Debts owed to the mortgagee, i.e. the lender, then 3. Any amount left goes to the mortgagor, i.e. the borrower.

A lender may seek a deficiency judgment against the borrower if the proceeds are insufficient to satisfy the loan, but the likelihood of this being enforced is slim.

Page 107: MBE Portion UBEBB

Titles A title in land is a bundle of proprietary rights that a party owns over the land in interest, which may be either legal or equitable. The guides I’ve been reading have been very direct in not going through the ins-and-outs of title, rather, they focus on the things you’ll need to know for the UBE, which is fair enough. So, that’s what I’m going to do. First up in this topic: ADVERSE POSSESSION, which is sometimes referred to as “Squatting”. Whilst that is one instance of adverse possession, it isn’t the only manifestation. Adverse possession can include things like a fence on the neighbour’s side of the lawn, or continuous use of a private driveway where no easement has been granted or prescribed. In short, adverse possession allows a possessor of land to acquire the title to that land. The possession must be:

• Continuously used for the statutory period by the possessor,4 • Open and notorious to the extent that it notifies the true owner, • Exclusive, meaning the true owner does not use it, • Actual, meaning that there is actual use of the land, and • Hostile and under the claim of right without the true owner’s consent, meaning

using without permission of the true owner. Adverse possessors in privity may aggregate their years spent possessing the property to meet the statutory period, which is known as aggregation and tacking. Constructive adverse possession is when a person takes possession of a portion of land under colour of title (this is where the alleged owner has some deed claiming to transfer title to them). The possession extends to the entire portion of land described in the aforementioned deed. DEEDS are legal instruments which affirm or confirm an interest, right or property. For a deed to be valid, it must:

• Be in writing to satisfy the Statute of Frauds, • Identify the land and the parties involved, and • Indicate the grantor’s intent to convey the land.

Intent to convey is presumed upon the construction of a deed but it will not be found if the grantor either expressly reserves the right to revoke the deed or the grantor gives an instruction to only deliver the deed upon the grantor’s death. If a deed is unconditional prima facie and given to the grantee as such, any oral conditions are invalid. However, if that same deed is given to a third-party, e.g. to be held in escrow, the additional conditions construct conditional delivery:

4 Note, however, this period applies only to the possessor, rather than applying to the length of time the true owner may have “dispossessed” the property.

Page 108: MBE Portion UBEBB

…deed… Description General warranty…

The most common type of deed and is used in everyday real estate transactions. They contain six covenants of title: Present, i.e. at the time of conveyance

• Seisin (the statement that the person is the rightful owner at present),

• The right to convey (that the person transacting has a right to convey the property), and

• A covenant against encumbrances Future Covenants

• Warranty to defend against any third-party claims to title, • Quiet enjoyment, i.e. to not be bothered by a third party’s claim

to title, and • Further assurances to do whatever is reasonable to perfect the

title.

Quitclaim… This is a deed that states the former owner quits ownership of the property therefore has no basis to claim an interest over the property.

Estoppel by…

If a grantor conveys a title to real property before they own it, the title will automatically vest in the grantee as soon as the grantor acquires title.

Wild… A deed not recorded properly and thus not discoverable in a chain of title. It does not put subsequent purchasers, i.e. purchasers after the wild deed was made, on constructive notice of a land transfer

Chain of title shows all transfers for a piece of land. As mentioned above, a wild deed brings unnecessary complications to the overall real estate transaction because it effects recordings of deed registrations. RECORDING ACTS are statutes which dictate the legal procedure by which an individual claiming an interest in land formally establishes their claim to that property. There are different rules for different types jurisdictions, which I will outline in a table below after going over the important terms:

• A bona fide purchaser is a purchaser who: o Takes real property without notice of a prior instrument, and o Pays valuable consideration; and o Is not a done, heir or devisee;

• The shelter rule states that the person who purchases land from a bona fide purchaser receives the same status and rights as a bona fide purchaser;

• Notice here refers to: o Actual notice, o Constructive notice, or o Inquiry/inspection/investigation notice.

Jurisdiction Rule Effect

Notice Statute A subsequent purchaser for value wins if, at present (time of conveyance), the subsequent purchaser

A subsequent bona vide purchaser will prevail over a prior grantee that failed to record

Page 109: MBE Portion UBEBB

had no actual or constructive notice

Race Statute Whoever records wins Whoever records first prevails; no requirement for notice

Race-Notice Statute A subsequent purchaser is only protected if they record before the prior guarantee

A subsequent purchaser is only protected if they record before the prior guarantee

Page 110: MBE Portion UBEBB

Torts Introduction Here we are, the end of the substantive content you’ll need to know for the MBE portion of the UBE. It also helps greatly that we are just about 60% done with learning substantive content for the UBE in its entirety, which is outstanding! As a bonus, one of the more interesting things about this is that at least one of the topics we’ll have covered by the end of this section will crop up as an essay question. A tort is a civil wrong committed by one or more legal persons against one or more legal persons. Like a crime is the word for a criminal wrong, a tort is the word for a civil wrong not covered by contract. Instead of jail-time, the remedy for a tortious Claimant may be in the form of damages, injunctive relief, or a combination of both. Unlike Contract Law, the damages within tort law can be punitive because punitive damages are meant to punish the Defendant.

Page 111: MBE Portion UBEBB

Intentional Torts As the name might suggest, an intentional tort is a civil wrong that is done purposefully. Intentional torts function differently to crimes in that there is no real “mens rea” so to speak; there is only the idea of intent meaning an assent to commit an action. Under the doctrine of transferred intent, it matters very little whether a Defendant intended to commit the wrong against someone specific, so the analysis for whether there has been an intentional tort is very straightforward. I’m going to put all the intentional torts in a table below:

Tort Description/Requirements Transferred Intent? (Y/N)

Remedy

Battery There must be: • Intentional, • Contact that the

reasonable person would find offensive and harmful, to

• The Plaintiff’s person or anything connected to the Plaintiff

Intentional here means that there is a desire to bring about harm or contact or knowing that harm or contact is substantially certain to occur

Y Damages

Assault There must be: • An intentional act, which • Causes, • The Plaintiff to be placed in

reasonable apprehension, of

• Imminent harmful or offensive contact with the Plaintiff’s person or anything connected to the Plaintiff.

Y Damages

False Imprisonment

This is when a Defendant commits:

• An intentional act, which • Causes, • The Plaintiff to be

restrained by physical force or through threats to boundaries, with

• No reasonable means of escape

Y Damages

Page 112: MBE Portion UBEBB

Intentional Infliction of Emotional Distress

The Defendant: • Acted intentionally or

recklessly, which manifested in

• Extreme and outrageous conduct that transcends all bounds of decency, which

• Caused extreme emotional distress, to the extent that

• The Plaintiff suffered actual and severe emotional distress

N Damages

Trespass to Land

The Defendant: • Intentionally, • Entered the Plaintiff’s land

either: physically or through the propulsion of physical objects

There is no intent to trespass required, the Plaintiff must only show that the Defendant intended to enter the land. The Plaintiff must also be the possessor of the land at the time of the entry.

Y Damages; Plaintiff can also recover a decrease in the value of the property or the cost to repair the property

Trespass to Chattels

The Defendant: • Intentionally interferes with

the Plaintiff’s personal property, and

• The amount of damage resulting from the interference is small

Y Damages only recoverable if the interference was substantial to amount to dispossession or if there has been an injury related to the tort

Conversion The Defendant: • Intentionally interferes with

the Plaintiff’s personal property, and

• The amount of damage resulting from the interference is substantial

Y Damages in an amount equal to the fair market value of the chattel

With intentional torts, it is generally easier to get punitive damages (damages that go above compensatory ones) because the objective of punitive damages is to punish the tortfeasor (the person committing the tort).

Page 113: MBE Portion UBEBB

There are also some general defences to intentional torts. The most common of these is consent, which may be express or implied through words. Importantly, though, the would-be tort cannot exceed the boundaries of the consent given and consent may be withdrawn at any time. If we consider a crime to be an offence against the State, and consent to a tort being consent to a would-be offence against the person, it makes sense that a would-be Plaintiff cannot consent to a crime. Another defence is privilege, which means that the Defendant commits an act that would ordinarily make them liable of an intentional tort but is excused under the circumstances. Regarding the trespass and property offences, there is a defence of necessity which means that the Defendant is not liable for harm to the Plaintiff’s property if their intrusion reasonably appeared to be or was in fact necessary to prevent serious harm to a person or property. Finally, there is a self-defence idea within intentional torts that sates the Defendant is not liable for harm to the Plaintiff if the Defendant reasonably believed that the Plaintiff was going to harm them (or another person), and the amount of force used to prevent this harm from occurring was reasonably necessary.

Page 114: MBE Portion UBEBB

Negligence Torts The torts you will be more familiar with are the negligence ones. These don’t require any intent to be committed and are very much grounded in the idea of what the reasonable person would (or wouldn’t) do in the circumstances. Generally, for someone to be liable for broad negligence, there must be:

1. A duty, which was 2. Breached by the Defendant, which 3. Caused the Plaintiff to suffer, 4. Damages.

A party must offer sufficient evidence so that a reasonable jury could find all of the above elements. The Duty, The Breach and The Reasonable Person Owing a duty is a matter of debate, or rather, to whom the duty is owed. Under Justice Andrews’ view, the duty is owed to all foreseeable Plaintiffs within a specific circumstance. This is a broader interpretation of Justice Cardozo’s view which states that the duty is owed to all foreseeable Plaintiffs within the zone of danger. In Part II of this guide, I’m going to properly undergo analysis of tortious issues and how this nuance will get you extra marks in the UBE. There is also the idea of an affirmative duty, i.e. a duty to act a certain way. In general, the “duty” that the negligence analysis refers to is an ongoing duty that a person already owes to the Plaintiff. Therefore, an affirmative duty to act in a certain (and mostly preventative) way does not exist except for limited circumstances:

• A pre-existing relationship between the parties, e.g. a parent-child relationship, or a person and their guest;

• A situation where the Defendant placed the Plaintiff in peril; • A situation where the Defendant has already undertaken to rescue the Plaintiff,

but liability only emerges if the Defendant’s action increased the risk of harm or harm is suffered by the Plaintiff on reliance of the Defendant’s action; or

• There is a duty imposed by law. Attached to the duty is the duty to act as a reasonable person would under the circumstances. But who is this “reasonable person”? This is a legal fiction used by jurists to help codify the ever-changing landscape of society and shifting scale of what we consider to be customary. The reasonable person is someone who follows the community customs and statutory requirements, but not to the extent that they are a perfect person. In short, the reasonable person is an objective standard. If the Defendant is a child, the duty imposed on them is to act as a hypothetical child of similar age, experience and intelligence would under similar circumstances. The exception to this, however, is if the child is engaging in adult activities. In this case, the child has a duty to act as a reasonable adult would. If the Defendant is a professional, their duty to act is in accordance with the knowledge and skill that an average member of that profession would act if participating in a similar community.

Plaintiff Reasonable Person Exceptions/Nuances Adult An objective standard to

act how a reasonably prudent person would act under the circumstances

Page 115: MBE Portion UBEBB

Child A duty to act as a child of similar age, experience and intelligence would act under the circumstances

If the child is engaging in adult activities, they must act how a reasonable adult would in the circumstances

Professional A duty to act with the knowledge and skill as an average member of that profession participating in a similar community

Specialist - They are held to the

standard of an average member of that profession practicing that specialty

Medical Doctor

- They are held to the national standard of an average qualified practitioner

The base case of a negligence tort is as described above: an owed duty of care that was breached to the extent that the breach caused damage. Now, I want to get into more specific examples, like for instance the landowner’s duty to entrants, which is a duty for a landowner to take reasonable steps to protect people that enter their property. In some states, this duty extends to all entrants, even illegal ones; however, in the majority of states, this duty varies dependent on who is entering the property. In these states, if:

• The trespasser is undiscovered, there is no duty; • There is an anticipated trespasser, i.e. a trespasser that is there but expected

to be there, there must be: 1. Reasonable care in any operations on the property, and 2. The owner must warn of or make safe any highly dangerous, artificial

(man-made) conditions that the landowner knows of; • There is a licensee, e.g. a social guest who has a reasonable belief of the right

to enter, there must be: 1. Reasonable care in any operations on the property, and 2. The owner must warn of or make safe any dangerous conditions of

which the owner knows but are not apparent to the guest; • There is an invitee, i.e. someone there on invitation of the owner and for the

owner’s benefit, there must be: 1. The same duty as a licensee, and 2. The owner must make reasonable inspections to find and make safe

non-obvious but dangerous conditions. As you can see, the duty of care owed is proportional to how well the Defendant would know the Plaintiff: the more cordial the Defendant and Plaintiff are, the greater the duty owed. Another negligence tort is called attractive nuisance which, though the name might suggest, is not the ordinary understanding of nuisance (as will be covered later). This

Page 116: MBE Portion UBEBB

is a tort which obliges a landowner to owe a duty of care to children who trespass on the property if the property contains things that may be attractive to them, e.g. a slide, a pile of tyres, etc. The landowner or the possessor of the land at the time has a duty to child trespassers to warn of or make safe artificial conditions on their land. They will be liable for attractive nuisance if:

• They know or should know of a dangerous artificial condition that is likely to cause death or serious bodily injury, and they

• Know or should know that children are likely to frequent the area, and that • The children are unlikely to discover the condition or appreciate the risks, and

the situation is so risky that • The risk of harm outweighs the expense of making the condition safe.

Importantly, the landowner is not liable if the child is engaging in adult activity, like driving a car for instance. Under tort law, landlords generally do not possess a duty of care to tenants. Though this may be remedied through contract law in the form of a signed agreement, there exists no duty at common law that a landlord must protect the tenant from the criminal acts of outside persons. There are instances where someone has broken a codified law and, in doing so, has breached a duty of care. Thinking back to the start of this section, I mentioned that there exists a duty if one is imposed by law; what I was alluding to was the idea of negligence per se, where the elements of a duty and breach of that duty are established through statute. For a Defendant to be liable of negligence per se, they must have breached a statute and:

• The statute’s purpose must have been to prevent the type of injury caused as a result of the breach, and

• The Plaintiff is in the class of persons that the statute seeks to protect. For example, breaking a law by running a red light and knocking a pedestrian would be an instance of negligence per se because the purpose of the law prohibiting cars from running a red light is to prevent the injury of running someone over, and the injured pedestrian is in the class of persons that the statute seeks to protect. There are two “exceptions” to this, but I would rather classify them as “challenges” from the Defendant’s side:

• Compliance with the statute would have been more dangerous than its violation, and/or

• Compliance with the statute was impossible. A broader interpretation of negligence per se is res ipsa loquitur, which means “the thing speaks for itself”. In effect, it means that negligence can be inferred from the nature of the action or injury without evidence of how the Defendant behaved. In other words, based off the details of the injury caused, negligence may be inferred even if a specific breach is difficult to ascertain. For a Plaintiff to rely on res ipsa loquitur, they must show that:

• The injury is a kind that typically does not occur without negligent behaviour, and that the

• Thing or object that caused the injury was in the Defendant’s exclusive control, to the extent that

Page 117: MBE Portion UBEBB

• Negligence was within the scope of the duty owed by the Defendant to the Plaintiff.

Causation We last discussed causation in reference to homicide, and the principle from there is virtually the same. The Plaintiff must show that the Defendant’s conduct was both the actual and proximate cause of the injury. “Actual”, meaning the “but for” cause; and “proximate”, meaning the legal cause, i.e. the injury must have been foreseeable to the Defendant as a result of their breach of duty. An intervening cause is an event that occurs after the breach but contributes to the harm. If the intervening cause was completely independent of the breach, thus unforeseeable by the Defendant, the Defendant would not be liable for the injuries caused by the intervening cause. If, however, there is a dependent intervening cause, i.e. one that arises solely as a result of the Defendant’s actions, this is usually a foreseeable effect and thus falls within the scope of the Defendant’s liability. Damage The Plaintiff must suffer actual damage, or harm, in order to bring a claim of negligence against a Defendant. This bar is extremely low, and the Plaintiff need not possess negative feelings towards the damage, though it is up for debate why they would even bring a suit in the first place if that were true. The point is, the damage or harm must be actual and real. From the Defendant’s side, as we mentioned before about the affirmative duty, there is a duty to “take the Plaintiff as the Plaintiff is found”, otherwise known as the “Eggshell Plaintiff Rule”. In effect, this means that a Defendant is liable for all harm that occurs to the Plaintiff as a result of the Defendant’s breach of duty, even if the Plaintiff’s own condition would worsen their resultant damage to a degree more severe than would a reasonable person. There are some defences to negligence torts, which I will put in the table below:

Defence Description, Requirements and Effect Pure comparative negligence

The Plaintiff’s own negligence will reduce the recovery of damages based upon the degree to which this negligence contributed to the injury.

Partial comparative negligence

If the Plaintiff is more than 50% at fault, the Defendant is not liable for negligence (or, rather, the Plaintiff’s claim is barred).

Contributory negligence

The Plaintiff’s claim is barred completely if they contributed to their injury. This is substantially harder to prove for the Defendant so is less likely to be used in court. There are some exceptions which will prevent this from being used by the Defendant, namely that the Defendant the last opportunity to avoid the harm or the Defendant themself was reckless.

Assumption of risk If the Plaintiff voluntarily assumed a known risk, their claim is barred. This assumption can be express, e.g. affirmatively stating that they assume the risk, or implied, e.g. conduct which indicates their assumption of risk.

Page 118: MBE Portion UBEBB

We so far have looked at negligence resulting in actual, physical harm, but there exists a negligence for the infliction of emotional distress. If you’re from the UK, you will know about the extremely specific cases regarding psychological harm, and the US is not too different. Still, it is worth knowing, not least of all because it may very well show up as part of a negligence question-set within the UBE. For emotional distress, there are only three scenarios in which a Plaintiff can recover damages:

1. A near miss case, where the Defendant was: a. Negligent, to the extent that there was a b. Foreseeable risk of injury to the Plaintiff, who was c. In the zone of danger, to the extent that d. The Plaintiff manifests physical symptoms;

2. A bystander claim, where the Defendant was: a. Negligent, and b. The Plaintiff was a contemporaneous witness to a negligent bodily injury

inflicted on someone close to the Plaintiff, like a family member, and c. The Plaintiff was in the zone of danger,5 to the extent that d. The Plaintiff manifests physical symptoms;

3. A pre-existing relationship claim,6 where there is: a. A pre-existing relationship between the parties, and b. The Defendant’s negligent act foreseeably causes distress to the

Plaintiff. Emotional distress is an extremely high bar; the manifestation of physical symptoms places a difficult threshold to prove. The symptoms must arise from the psychological harm and can include sleeplessness, fainting and getting concussed, throwing up, but a good Defendant would claim that they are not a proximate cause for the symptoms. So far, we have considered negligence in the form of a Defendant committing some breach against a singular Plaintiff. The scope of torts, however, is such that the doctrine of respondeat superior pervades the discourse to enable Plaintiffs to bring actions against a company for their employee’s negligence. Respondeat superior literally translates to “let the master answer”, meaning that the “master” is legally responsible for answer for the actions of people under their purview. I’m doing away with the word “master” because it’s awful, but you might be able to see how this is applied in a contemporaneous context when considering employer/employee relationships, for example. Under this doctrine, an employer is liable for an employee’s negligent acts if the employee was acting within the scope of the employment, meaning performing work assigned by the employer or engaging in a course of conduct that was subject to the employer’s control. In analysing the limits to the “scope of employment” requirement, the courts will evaluate whether:

• The conduct was of the kind the employee was employed to perform, • It occurred substantially within the authorised temporal or spatial limits, and • It was motivated to serve the employer either in whole or in part.

Therefore, conduct is not in the scope of employment if it was unrelated to the employment and not intended to serve any purpose of the employer, except for in the

5 Some states require this, but most don’t. 6 Super rare to recover for damages

Page 119: MBE Portion UBEBB

following circumstance, where an employer will be liable for an employee’s acts outside the scope of employment if:

• The employer intended the conduct, and • The employer was negligent or reckless in selecting, training, supervising, or

controlling the employer, in the performance of a • Non-delegable duty, or alternatively • The agent (person committing the tort) had apparent authority upon which a

third-party relied. Bear in mind, however, this is only the rule for negligent torts; generally, an employer cannot be liable for intentional torts committed by their employee except in a few very strict circumstances:

• The intentional tort was expressly authorised by the employer, • The intentional tort was a natural product of the agent’s duties, or • The intentional tort was motivated by a desire to serve the principal (the

employer, in this instance).7 The analysis gets a little bit murkier when we look at the difference between an independent contractor and an employee. When I say murkier, I mean generally a lot more complex because whilst someone can present themselves as an independent contractor (and indeed someone seeking to escape vicarious liability would want to do so) the court cares very little about the technical definition when determining liability. The general rule is that an employer or principal is not liable for torts committed by an independent contractor, save for the following exceptions:

• The work being done by the contractor is inherently dangerous, • There is a non-delegable duty owed by the principle, meaning that the principal

cannot offload their duty to someone else, or • Estoppel in which the principal holds out the contractor as an agent, a third-

party relied on this skill, and the reliance on the skill caused harm to the third-party.

However, in determining whether someone should be treated as an independent contractor as a matter of law, the court applies the following rubric to analyse:

• The type of work being done, • The pay-rate, • Who supplied the equipment and tools, • The degree of supervision, • The degree of skill required, • Whether the work was for the benefit of the employer’s business, • The extent of control that the principal has over the work details, and • Whether the contractor is engaged in a distinct business.

Here’s a table which should show how an employee versus a contractor would deal with the above:

Issue Contractor Employee Type of work

If the work done is removed from the function of the business, they are likelier to be a contractor

If the work done is more proximate to the function of the business, they are likelier to be an employee

7 See Agency in Part II(8) (MBE: Agency)

Page 120: MBE Portion UBEBB

Pay If the person is paid daily/weekly and receives no benefits from the employer, they are more likely to be a contractor

If the person is paid on a monthly basis and receives some benefits from the employer, they are more likely to be an employee

Supplied equipment and tools

If the person supplied their own equipment and tools, they are more likely to be a contractor

If the employer supplied their own equipment and tools, they are more likely to be an employee

Degree of supervision

If the person carried out the work largely unsupervised, they are more likely to be a contractor

If the person carries out the work largely supervised, they are more likely to be an employee

Degree of skill required

If the skills required for the job are outside of the degree of skill contained by the average employee at the business, they are more likely to be a contractor

If the skills required for the job are within the degree of skill contained by the average employee at the business, they are more likely to be an employee

Benefit of the employer’s business

If the work done has little substantive benefit to the employer’s business, they are more likely to be a contractor

If the work done has a substantive benefit to the employer’s business, they are more likely to be an employee

Extent of control exercised by the principal

If the principal has strong control over the work, they are more likely to be a contractor

If the principal has little control over the work, they are more likely to be an employee

Distinct business

If the person doing the work works in a business that is distinct from the principal’s, they are more likely to be a contractor

If the person doing the work works in a business that is not distinct from the principal’s, they are more likely to be an employee

Page 121: MBE Portion UBEBB

Tortious Liability of Multiple Defendants Real life is often more complex than a restrictive assignment that asks you to focus on the tortious liability arising from one single Defendant. A truer-to-life scenario contains multiple Defendants, and it is our job here to discuss what this means in a liability context. For starters, let’s dissect what we mean by “liable”. Tortious liability means that the Plaintiff has effectively shown that a particular party (or parties, as we will note) owed a duty of care, breached that duty, and that breach of duty caused harm. If someone “caused” harm, it means both factual and proximate (legal) causation. The analysis for factual causation is a much lower bar than for proximate, and the law generally doesn’t want to punish people unnecessarily; therefore, if there are multiple Defendants, the Plaintiff must show that the multiple Defendants are the proximate cause of a single, indivisible harm. This creates joint and several liability amongst the Defendants, meaning that the Plaintiff can recover the entire amount of damages from a single Defendant since their respective liabilities have been conflated. Any Defendant who pays more than their share of damages to the Plaintiff may sue the others for contribution. There are also scenarios in which the true singular actor is difficult to identify amidst a group of Defendants. In these instances, the doctrine of alternative liability allows a jury to find all the Defendants liable if:

• Multiple Defendants were negligent, but • It is unclear which one caused the Plaintiff’s injuries.

Another potential form of tortious liability of multiple Defendants comes in the form of the doctrine of joint enterprise. Fellow UK graduates, remove your instincts because this is a term which means exactly what it says and it has no relevancy within Criminal Law for the UBE, only Torts. Under the doctrine of joint enterprise, the negligence of one Defendant can be imputed to other Defendants if:

• The multiple Defendants were engaged in a common project or venture, and • All the Defendants made an agreement to engage in what would become the

tortious conduct. The final example to note refers to product liability (which will be discussed in the next section), but it’s just easier to get this all in one place. The doctrine of market share liability is where all manufacturers of the product identical to the one that harmed the Plaintiff are liable in proportion to their share of the market. For this to apply:

• All the Defendants must be potential tortfeasors, who all create • An identical product with the same defect, and through no fault of their own • The Plaintiff is unable to identify which Defendant caused the injury, therefore • All manufacturers of the product are named as Defendants.

Finally, a passive tortfeasor can indemnify themselves, meaning they can assert a claim against an active tortfeasor to recover the full amount that it paid to the Plaintiff. If you think back to CivPro, this would be an example of a crossclaim. This is different to contribution, in which there are multiple active tortfeasors but one has overpaid and thus seeks a contribution from the other Defendants to make up the lost funds.

Page 122: MBE Portion UBEBB

Strict Liability Torts A strict liability tort is a tort which differs from intentional torts and negligence torts because there is no “fault”. With an intentional tort, the intent to commit harm is the fault. With a negligence tort, the breach of the duty causing the harm is the fault. With a strict liability tort, the Plaintiff need only show that harm occurred, and the Defendant was responsible for it. There are three strict liability torts to know for the UBE:

• Torts involving animals, • Torts involving abnormally dangerous activities, and • Strict products liability.

Torts involving animals are fairly straightforward. Generally, the owner of a domestic animal is not liable for the animal’s actions unless they have knowledge of the animal’s vicious tendencies. The owner of a wild animal (note, this is not an “animal in the wild” which has no owner) on the other hand is strictly liable for any harm that occurs regardless of any safety precautions taken to prevent harm from occurring. An abnormally dangerous activity is defined as an activity that:

• Is not of common usage in the community, and to the extent that it • Creates a foreseeable and highly significant risk of physical harm even when

reasonable care is exercised. A Defendant will be found strictly liable if their engagement in an abnormally dangerous activity resulted in harm to the Plaintiff. And, finally, strict products liability. The rule is that a commercial supplier is strictly liable for any harm caused by its products. A commercial supplier is defined as any person or entity engaged in the business of selling goods of the type, i.e. they regularly sell the questioned goods, meaning that ad-hoc sellers or service providers are not commercial suppliers for the definition of strict products liability. In order to create the liability, the Plaintiff must show that:

• The product was defective, which can be a manufacturing defect, a design defect, or a failure to warn of a defect, and

• The product was not altered when it reached the Plaintiff, which • Caused an injury to the Plaintiff when it was used in an intended or unintended

foreseeable use, which resulted in • Damages such as personal injury or property damages, not solely the economic

loss of the price of the product, and • The Defendant is a commercial supplier engaged in selling goods of the type.

A manufacturing defect means that the product differs from others of the exact type and is therefore more dangerous than if made properly. A design defect means that the product is not designed as safe, as practically or as cost effectively, which therefore creates a risk to the end consumer. A failure to warn is where the Defendant does not warn the Plaintiff of the risks, which themselves must be un-obvious to ordinary users, but known to the designer or manufacturer; the warning should be, at the very least, proportionate to the risk in question.

Page 123: MBE Portion UBEBB

MIND Torts There are some torts which are difficult to categorise due to the fact that they don’t wholly align with intentional or negligence torts but cause less actual harm than strict liability torts. Some guides call them the “Economic” torts, but I think that’s stretching the definition of what could be an economic loss very thinly. After all, it can be argued that all torts are economic since the sought remedy is damages. I’ll alter this and call them the MIND torts, i.e. torts involving:

• Misrepresentations, • Interferences, • Nuisances, and • Defamation (and privacy).

A MISREPRESENTATION is defined as a statement which causes a person to think something different to what the true nature of a thing is. There are two types of misrepresentation in torts: intentional misrepresentation and negligent misrepresentation. An intentional misrepresentation is also called fraud or deceit and the Plaintiff must show that the misrepresentation was:

• A misrepresentation of a material fact, and that the • Defendant knew the statement was false, but made it anyway, with • The intent to induce the Plaintiff into an engagement, which caused the

Plaintiff to • Have actual and reasonable reliance, but nonetheless, the Plaintiff suffered • Damages.

In general, there are no defences to intentional misrepresentation save for the Plaintiff signing a contract which excludes their reliance on the statement, in which case the Plaintiff has effectively “consented” to hearing the statement therefore there is no tort committed. The other type of misrepresentation is a negligent misrepresentation which requires the Plaintiff to show:

• A false statement of material fact, which was • Supplied for the guidance of others in a business transaction, and • The Defendant knew or should have known that the information was supplied

to guide the Plaintiff in their transactions, yet • The Defendant was negligent in obtaining or communicating the information,

which caused the Plaintiff to • Have actual and reasonable reliance on the statement, to the extent that • The statement proximately caused the Plaintiff to suffer damages.

The ordinary defences to negligence apply here. An intentional tortious INTERFERENCE with business relations is defined as an instance where one person intentionally damages someone else’s business relationship with a third-party. The Plaintiff must show that:

• There was a valid contract or business relationship with a third-party, and • The Defendant knew of this, but nonetheless • The Defendant intentionally induced the third-party to breach the contract or

agreement, to the extent that • The breach occurred, causing • The Defendant to suffer damages.

Page 124: MBE Portion UBEBB

The next torts to cover are the NUISANCE torts, namely: public nuisance and private nuisance. Public nuisance is an act that obstructs, damages or inconveniences the community. The Plaintiff must show:

• There was an unreasonable interference, with • The health, safety or proper rights, of • The community, which caused • Actual damage.

On a more personal side, private nuisance deals with the interference of an individual’s use and enjoyment of their property. The Plaintiff must show:

• There was a substantial and unreasonable interference, with • The Plaintiff’s use or enjoyment of their property, that • The reasonable person would find offensive, inconvenient or annoying.

Finally, let’s discuss the DEFAMATION (and privacy) torts. These are torts where the economic impact is generally nil or not fully realised until potentially much, much later. Defamation is something you’re familiar with; a statement about someone that causes them to be defamed, i.e. the perception of their reputation has been reduced. There are two types of defamation: slander and libel. Slander is an oral defamatory statement, whereas libel is embedded in a permanent format. The table below highlights some examples of the differences:

Slander Libel Saying something false and negative about someone to someone else

Writing a article about someone which contains false and negative allegations Writing a false and negative comment about someone on a blog or social media Making a false and negative claim about someone in a book

To actually commit defamation, there must be:

• A defamatory (negative) statement, • Of and concerning the Plaintiff, made by the Defendant, and this was • Published by the Defendant to a third-party, causing • The Plaintiff to suffer damages.

If the Plaintiff is a public figure, the Plaintiff must also prove that the statement was false and the Defendant had actual malice, i.e. recklessness or knowledge that the statement was false, when they publicised it. If the Plaintiff is a private figure but the statement has a public concern, the Plaintiff must also prove that the statement was false and the Defendant committed negligence in publicising the statement. To claim damages for slander, the Plaintiff must show special damages, i.e. the exact financial impact that the temporary statement had on their reputation. To claim damages for libel, the Plaintiff does not need to show special damages unless the statement itself is merely allusive. Slander per se is actionable without proof of special damages and will apply when the statement concerns any of the following issues:

• Imputation of a business integrity or skill, • The unfaithfulness of a married woman, • A loathsome disease, or • A crime of moral turpitude.

Page 125: MBE Portion UBEBB

Of course, the defence for defamation is truth, which completely bars the Plaintiff’s action. And, finally, the invasion of privacy torts… I wonder what they concern. Of course, they concern the invasion of privacy by one person of another. This is an important violation because in almost every jurisdiction in the world, there is some consensus on the belief that human beings have inalienable rights to privacy. I’m going to put these in a table because there aren’t any strict nuances of these, they’re rather rote.

Tort Description and Requirements

Specific Details Defences

Misappropriation of Name or Picture

The Plaintiff must show that the Defendant:

• Used the Plaintiff’s name or likeness, for

• Commercial advantage

--- Newsworthiness

False Light The Defendant: • Caused

widespread dissemination, of

• The Plaintiff’s beliefs, thoughts or actions, which were shown in a

• False light, • That would be

objectionable to the reasonable person

If the Plaintiff is a public figure or the information is a matter of public concern, the Plaintiff must show the Defendant acted with actual malice, which has the same definition as above

Truth, but only if the implication of the Defendant’s statement were true a

Intrusion of Privacy

The Defendant: • Intrudes into the

private affairs of the Plaintiff, who

• Has a reasonable expectation of privacy, and

• The intrusion is highly objectionable to the reasonable person

Consent and Privilege

Public Disclosure of Private Facts

The Defendant must have:

• Caused the widespread dissemination, of

If the Plaintiff is a public figure or the information is a matter of public concern, the Plaintiff must show the Defendant acted with

Consent but never Truth

Page 126: MBE Portion UBEBB

• Truthful, private information about the Plaintiff, that is

• Highly objectionable to the reasonable person

actual malice, which has the same definition as above